Monday, April 09, 2012

A defense of the First and Second Ways

HT: Ben Yaachov

243 comments:

1 – 200 of 243   Newer›   Newest»
Son of Ya'Kov said...

Good Luck getting BI to actually read it.

Anonymous said...

Thanks for posting this. I will look into it more. I think I'm going to go along with the "morally sufficient reason" thingy with the problem of evil and suffering. The God I believe in is in control of everything. When I say that God is in control of everything, I don't mean that He directly causes everything. For this would make Him the author of evil. Rather, He uses secondary causes by permitting evil and suffering (for morally sufficient reasons) to bring about His overall plans and purposes. It is precisely because He is Holy and in control that He will come through on His promises that have been certified by His loving work at the cross. This is what gives me hope. When my faith is in Him my future is secure and anxieties are broken as my heart opens up to love others in the present. Faith - Hope - Love. Just knowing that God desires all to be saved and His fierce anger will not turn back until He accomplishes the purposes of His heart gives me hope. Indeed, He will reconcile all things to Himself and no plan of His can be thwarted. In the end all will confess Christ is Lord to the glory of God and be saved.

Son of Ya'Kov said...

I am skeptical this will do Cole any good (since in just a short while he was an Atheist. Now he believes again. Well I don't think God holds his illness against him.) but I am going to recommend he read THE REALITY OF GOD AND THE PROBLEM OF EVIL by Brian Davies.

Also if he has the time THOMAS AQUINAS ON GOD AND EVIL also by Davies.

THE LAST SUPERSTITION by Feser might help too.

OTOH in a week he could go back to being an Atheist regardless of what he reads & then a month later flip again.

But I give what help I can.

Shackleman said...

I cannot begin to appreciate the difficulties that must accompany a diagnosed mental disease, but my heart goes out to those who suffer in that way.

I can however identify with the sort of tug-of-war that accompanies the journey from atheism to theism, and then again to Christianity in particular.

Cole, it very well may be that you swing back, on occasion throughout your spiritual journey, to atheism. I have, and even now can sometimes feel myself sliding back down that cliff and it can sometimes take quite a bit to hold on...and I thankfully don't have the extra burdens that must come with your disease. Do not let this tug-of-war discourage you. I encourage you....Seek the truth. Pursue your Creator. Foster your trust, understanding, and love of Him. He will not forsake those who earnestly seek Him, but I've learned through my experience that that doesn't mean He will always seem near. And when He seems far, pray.

May the peace of Christ be with you, Cole.

B. Prokop said...

For myself, I require no proofs. But for those who like such things, here are my own "Five Ways":

1. There exists something rather than nothing.

2. The universe (and time) had a beginning.

3. Meaning, purpose, and beauty exist.

4. Good and evil are real.

5. Christ verifiably rose from the dead on Easter, after having beyond question died on Good Friday.

I have at least eleven more "Ways" besides these, but far be it from me to one-up Aquinas (or would that be eleven-up?)

Walter said...

As a deist I like Bob's third and fourth points. These are the main two that brought me back from atheism.

BeingItself said...

(1) Everything that is changing is being changed by something else.

That's empirically false.

Son of Ya'Kov said...

BI you didn't read the article.

You are simply nay-saying not offering an argument or even addressing the article's specific claims.

Heck your not even dragging out Hume's old worn out arguments (long dispatched by GEM Anscombe) to address the issue. Heck I doubt you could articulate any of them anyway.

In short ya got nothing.

So what are you gonna do Gnu?

Stamp your feet & say "Make an argument BenYachov you coward!"

(Well I did actually I argued you confused motus with momentum. As well as your confusing physics with metaphysics.)

While you refuse to read an academic paper right in front of you.

What's the matter is it to far above your head?

That would explain much.

(This is the part where you rant it's all confusion, language manipulation and sophistry and it's not scientific blah blah blah...)

Gnu Plueez!

Son of Ya'Kov said...

>>(1) Everything that is changing is being changed by something else.

>That's empirically false.

BTW any examples? Or are you just going to cite the phenomena in a Quantum Vacuum & work your Atheism-of-the-Gaps mojo?

BeingItself said...

Ben,

Things move (ie. change position) without anything pushing them. I realize this violates your naive notion of physics. But folk physics is false.

Things change without "being changed by something else".

The argument is unsound because the first premise is false.

Crude said...

Things move (ie. change position) without anything pushing them. I realize this violates your naive notion of physics. But folk physics is false.

You may want to actually read the paper, man.

Son of Ya'Kov said...

Oh BI you are so void of the ravages of intelligence. I think Linton should watch out you are clearly bucking for his job.

I know this is pearls before swine but here we go.

>Things move (ie. change position) without anything pushing them. I realize this violates your naive notion of physics. But folk physics is false.

So basically you ignored everything I wrote & have gone back to your modus=momentum meme & treating the argument as one from physics rather than as a metaphysical description of change i.e. of potency being reduced to actuality?

Also it's clear by your statement you still haven't read the paper.

>Things change without "being changed by something else".

You have still given no example not even in physics.

The ancients incorrectly concluded that physically moving objects need something to keep moving them or they would revert to their natural state of stasis.

This was incorrect.

Newton showed an object in momentum would stay in momentum unless acted on by an outside force.

Thus disproving the ancient folk physics (of course Einstein overthrew Newton and Heisenberg cast down Einstein but that is not important right now).

Now if we may look at the actual argument (which I take from Brian Davies' and Brian Leftow's translation of the Summa) instead of your self-serving ignorant paraphrase.

It is certain, and evident to our senses, that in the world some things undergo change. Now whatever is in the process of change is put into change by another, for nothing can be undergoing change except it is in potentiality to that towards which it is in change; whereas a thing changes inasmuch as it is in act. For change is nothing else than the reduction of something from potentiality to actuality. But nothing can be reduced from potentiality to actuality, except by something in a state of actuality.

Thus for your weird claim "that's empirically false" to be correct you have to give me an example of an object in momentum that changes it's momentum without being acted on by an outside force.

Oh & BTW parts of an object can move/change a whole object.
Keep your fallacy of composition to yourself.

Aquinas said in a sense animals can move themselves but in reality parts of an animal move other parts.

So giving me an example of a ball thrown in Zero gravity with mini-retro-rockets on it to change it's momentum as an example of it not being changed by another but itself is not legit.

Epic fail Gnu!

Son of Ya'Kov said...
This comment has been removed by the author.
William said...

ben: radioactive decay is not triggered by an outside force.

Anonymous said...

Ben,

Thanks for the resources. I think I'm here to stay this time. One of the reasons I have come back to Christ is because of C.S. Lewis and his ideas about beauty. For Lewis there seems to be an instinct for the transcendent that is awakened by beauty. Beauty stirs a longing for the transcendent. This seems to be true in my life. I have had intense experiences with what I consider to be "beauty". So intense that it felt like I was in God's presence. It just seemed like something was telling me that there has got to be more to it than just this finite existence. Here's one of those experiences that I recorded:

In the presence of Beauty I find the gentleness of mystical love. It is here that wonder is awakened and the beauties of humility, compassion, and kindness come alive. Within this matrix my fears melt away. I tremble at the thought of being inside this glowing radiance, for the longing of my heart is for a union with Beauty. To drink it in and become one with it. For me, Beauty brings warmth. Something in my soul longs for this delight, for Beauty inspires and delights my soul. When Beauty whispers Her tenderness to my soul I am filled with a childlike wonder and awe.

This is one of the things that I believe Christ was doing at the cross. He was removing God's wrath from my vision so that I could see and savor Divine Beauty in His own face. At the cross I was united to Christ in a mystical union. My sins were extrinsically transferred to the innocent Christ as God's wrath was poured out and Christ removed God's wrath from my vision as He died and was resurrected and then the righteousness of Christ was extrinsically transferred over to me. That is, I have died to the old self as it was crucified with Christ and clothed myself in the robes of Christ's righteousness as I am risen to new life. I am one with Christ in mystical union with Him. There is now no condemnation for me. For all that has been removed by Christ as I am in Him and He is in me. Death has been overcome. I can now experience the glory of God in the face of Jesus Christ.

B. Prokop said...

Since we're sticking our toes into the world of physics a bit in this thread, I would like to use this forum as an opportunity to publicly repudiate everything I wrote 40+ days ago on a previous thread about information and subatomic particles.

Much to my surprise, I've spent a fair piece of the last month or so reading up on quantum mechanics, guided by some very good friends of mine who have forgotten more about the subject than I will ever know.

It seems that my question "Where is the information stored?" at the fundamental particle level should have perhaps been expressed the other way round. For if I understand what I've been reading (and hearing) of late, there just may be nothing but information at the foundational levels of matter. The real question may actually be "Where is the matter?"

But at any rate, either question (let alone their answers) is a death knell to materialism. Either the entire universe in its most fundamental essence is non-material, or you still have the problem of non-material information guiding the actions of matter at the smallest scales.

Selah.

Son of Ya'Kov said...

>ben: radioactive decay is not triggered by an outside force.

Neither does a Super Nova happen by outside forces but because it burns up most of it's hydrogen & starts fusing helium plus having more mass then our sun.

What part of "parts of a thing moving other parts of a thing" is unclear to you?

Enough of the mechanistic bullshit & ignorance!!

Go read up on the first way starting with Feser or I will rag on you William.


Cole,

I'll pray for you guy.

Bob,

Welcome back buddy.

Son of Ya'Kov said...

Sorry William I just woke up & I am a little crabby.

BeingItself said...

Ben,

Just so I follow you, is the first premise of the argument now:

(1) Everything that's momentum is changing is being changed by something else.

Son of Ya'Kov said...

No it isn't.

BI either read the paper or go f*** off.

Pearls before swine like I said.

What a coward you can't read a simple academic paper.

BeingItself said...

Ben,

I've read the paper. The first premise if false.

You tried to defend it by moving the goal post, and when I call you out on it you just call me names. Grow up.

I really think you could benefit from exploring the critical thinking literature. Look into it.

finney said...

Welcome (back?) to the fam, Cole.

William said...

Ben: in order to accept the idea that argument 1 has no empirical, quantum mechanics based exceptions, one must first accept the metaphysics of Aristotle and Aquinas, which is actually a bigger pill to swallow for most moderns than the thing you want to prove (that God exists).

Of course I understand that from your perspective perhaps accepting God's existence and accepting Aquinas' philosophy may be one and the some thing. Forgive my partial atheism from your point of view :(.

B. Prokop said...

I will go out on a limb here and back up William's posting. Aquinas's argument as it stands is predicated on an acceptance of Aristotelian Physics. Today we no longer take Aristotle to be the last word on "How the World Works". This does not in the least invalidate Aquinas's logic, but it is a severe handicap in trying to shoehorn that logic into contemporary physics. There are just too many ill-fitting pieces.

But in the end that's no Big Deal. After all, Aquinas himself arose to prominence as a consequence of Aristotle's thought bursting onto the Medieval scene. All that is needed today is a 21st Century Aquinas, competent (I certainly am not!) to make Thomas's arguments up to date - capable of being expressed in a manner that speaks the language of quantum mechanics.

William said...

Bob,

I agree, and I think the presuppositions of current physics regarding natural laws would be where a modern Aquinas would start: natural laws are the closest thing we have to a modernist metaphysics (see the discussion of natural laws in the article).

BeingItself said...

I am puzzled by the last three comments.

Why does the argument need updating?

Are the premises true or not?

Supposedly all of you guys believe Yahweh exists because of these arguments.

Otherwise, you are just making up arguments to bolster what you believe for other reasons.

Which IMO is what apologetics is all about anyway. But you lot are not supposed to admit that.

William said...

Being: At risk of misinterpretation, I would says that Aquinas and Aristotle need updating because Aristotle attributed the regularities of natural things (his physics and metaphysics) to intrinsic properties of the individual objects involved, whereas modern physics is more holistic in some ways, and attributes many physical properties of objects to the properties of their surrounding space-time framework.

For example, Aristotle would say that an object falls simply because it has an intrinsic tendency to move that way, not because of any spacetime curvature in its vicinity.

Son of Ya'Kov said...

>I've read the paper. The first premise if false.

You are clearly lying. Where does Oderberg defend "folk physics"?

He doesn't you didn't read the paper & you have nothing to contribute here.

Goodbye!

Walter said...

Supposedly all of you guys believe Yahweh exists because of these arguments.

Not all of us believe in Yahweh.

Philosophical arguments of this nature are designed to foster belief in the monotheistic God of classical theism. Belief that Jesus died for your sins requires a whole other set of arguments that attempt to convince you that a divine revelation has occurred.

Son of Ya'Kov said...

@Bob

>I will go out on a limb here and back up William's posting. Aquinas's argument as it stands is predicated on an acceptance of Aristotelian Physics.

I will saw off that limb 100% incorrect Bob. Aquinas' argument is based on metaphysics not physics. The word "Motus/movement" in Aristotle and Aquinas refers to "change" not momentum or literal physical movement from point a to point b. Oderberg, Davies, Feser etc all show that.

Did you read the paper Bob because it doesn't appear you did?

Again Motus/motion refers to change(i.e. the reduction of something from potentiality to actuality). Aristotle was answering Parmenides(who btw held the same faulty folk physics as Aristotle) who claimed change was impossible and that our senses should not be trusted when they sense that things change.

Parmenides believed like Aristole that an object appeared to move from point a to b as long as something appeared to move it(bad physics).

But his metaphysics told him that real change was impossible and an illusion. So in the end the object didn't really move from point a to b regardless of the actual physics. It only appeared to do so.

If I where to translate Parmenides into Newton he would say it only appeared that an object in momentum stayed in momentum till acted on (i.e. changed) by an outside force. For him any change was unreal. Aristotle argued change was real. The 1st way is an argument from change not literal physical motion.

Oderberg explains that & so does Feser and other neo-Aristotilans.

The idea Aquinas' 5 ways is based on physics is up there with the claim Galileo "proved" the Earth went around the sun & you and I know from our past cordial discussion with Jesse that was incorrect.

It's the same here brother.

William said...

3 3 1+ - _
T -> He + e + v
1 2 e

The above is not about motion, Ben, but it is about spontaneous probabilistic change.

B. Prokop said...

To BeingItself:

I at least did not say that the argument needed updating. I most likely did not express myself clearly enough, but what I had meant to say was that Aquinas's argument is couched in terms (Aristotelianism) that do not translate easily into modern physics. It needs to be re-expressed in a 21st Century framework. The reasoning does not have to be altered, but it must be intelligible to its audience. As it is, reading through Aquinas's "First Way" today is like reading Beowulf in Old English.

On a related note, you write, "Supposedly all of you guys believe Yahweh exists because of these arguments." Not so. I refer to my above posting on this thread, in which I said, "For myself, I require no proofs." I meant that. Ben may call me a heretic for saying this (if he knows his history well enough to do so), but in my view, if it can be proven, it ain't God.

Which does not in any way mean that I believe one cannot find every tenet of the faith to be in perfect accord with reason. Indeed I maintain that there is no conflict whatsoever between reason and faith. The two are in complete harmony.

But to demand "proof" is to fall for the second temptation of the devil (second in Matthew's version of the encounter). I take Jesus at His word. The only sign this (or any) generation will ever get is the Sign of Jonah. The Faith stands or falls on the FACT of the Resurrection. Everything else is ultimately a side issue.

Son of Ya'Kov said...

Thank you Walter that was a good point but BI is IMHO a troll. He clearly hasn't read the paper. He does not understand the subject matter & he is not open to learning it. He just wants to argue by heckling.

@William

>Ben: in order to accept the idea that argument 1 has no empirical, quantum mechanics based exceptions, one must first accept the metaphysics of Aristotle and Aquinas, which is actually a bigger pill to swallow for most moderns than the thing you want to prove (that God exists).

You not making the mistake of assuming you come to this without even an implicit set of metaphysical assumption are you?

Because I submit you are unconsciously as well as uncritically using the Post-enlightement Mechanistic Metaphysics of Descartes, Kant and Hume. Why do you except them & have you ever read a neo-classical critique of their claims or read a defense of the classic metaphysics? I recomed it. It changed my whole outlook.

>For example, Aristotle would say that an object falls simply because it has an intrinsic tendency to move that way, not because of any spacetime curvature in its vicinity.

Rather if we updated Aristotle's physics but kept his metaphysics we would say Gravity has an intrinsic tendency to pull things toward it or that things with Mass have an intrinsic tendency to have gravity. The metaphysics are still valid.

>I would says that Aquinas and Aristotle need updating because Aristotle attributed the regularities of natural things (his physics and metaphysics) to intrinsic properties of the individual objects involved, whereas modern physics is more holistic in some ways, and attributes many physical properties of objects to the properties of their surrounding space-time framework.

Your equivocating of faulty physics with metaphysics is neither fair nor rational.

After all you are looking at the world threw the metaphysics of Descartes, Kant, and Hume. You filter out their limited and faulty views based on Newtonian physics while adapting them to the updates of Einstein and Heisenberg. Don't you?

So why is Aristotle so special? Aristotle is better.

Son of Ya'Kov said...
This comment has been removed by the author.
Son of Ya'Kov said...

So both Bob and William are wrong about Aquinas and Aristotle since they conflate physics with metaphysics and they Ad Hoc claim Aristotle's metaphysics can't be separated from his physics(without any argument or proof). Yet they don't realized they are using the Post-enlightenment Mechanistic Metaphysics of Descartes, Kant and Hume that is separated from these philosophers own discredited views on Physics.

Son of Ya'Kov said...

But I don't doubt the sincerity of either of you guy unlike BI who of course has made no effort to engage the argument from change.

Son of Ya'Kov said...

@William

>The above is not about motion, Ben, but it is about spontaneous probabilistic change.

However for you to show that change is truly "uncaused" (i.e. not caused by another) you have to show a change that occurs without sufficient conditions and without necessity.

Yeh good luck with that William.

William said...

"Nihil enim movetur, nisi secundum quod est in potentia ad illud ad quod movetur, movet autem aliquid secundum quod est actu.
"

I see nothing about some kind of disembodied necessity itself here, just about physical things and their interactions, and a need for a second thing to explain a first things's change.

But you want to argue your re-interpretation of the above words, of course. I cannot then disagree with you.

Son of Ya'Kov said...

You lost me William. Are you arguing against the 1st way or not? Are you claiming the 1st way is about physics or not?

Son of Ya'Kov said...

>Nihil enim movetur, nisi secundum quod est in potentia ad illud ad quod movetur, movet autem aliquid secundum quod est actu.

The first way in Latin.

For change(i.e. motion) is nothing else than the reduction of something from potentiality to actuality.

>I see nothing about some kind of disembodied necessity itself here, just about physical things and their interactions, and a need for a second thing to explain a first things's change.

Well what does "spontaneous probabilistic change" have to do with anything regarding the first way? Or to put it another way what does spontaneous probabilistic reduction of something from potentiality to actuality have to do with anything?

>But you want to argue your re-interpretation of the above words, of course. I cannot then disagree with you.

Your making a classic mistake. You are reading your modern terminology back into ancient terminology & falsely assuming they are equivalent.

I had this problem over at Feser's blog. This bright sincere Atheist dude was scandalized when he read Oderberg's paper on Hylomorphism dualism and complained "Oderberg rejects the idea we are made up of Atoms!"

I explained Democretus the ancient Atheist philosopher hypothesized all matter was made up of particles which where so small they contained no void. Now what we call Atoms today, units of matter made up of Electrons, Protons and Neutrons by definition are not "Atoms" as originally envisioned by Democretus. Since such atoms are made of smaller particles & thus by definition have void.

Indeed if there is anything in physics that resembles Democretus' "Atoms" I would nominate a Naked Singularity.

Well he went back re-read the paper and saw that I was right. Oderberg wasn't denying we are made up of Atoms in the Electron Protons etc..sense.

For Aquinas motions/movetur/Motus refers to the reduction of something from potentiality to actuality which is change at the fundamental level.

Those are the brute facts.

William said...

After internet search, I find that nothing I've said is original, so I give up after these quotes:

see bottom of:
http://edwardfeser.blogspot.com/2010/08/edwards-on-infinite-causal-series.html

and see:
http://www.trinity.edu/cbrown/intro/cosmological_argument.html

Papalinton said...

A must see video:

http://debunkingchristianity.blogspot.com.au/2012/04/peter-boghossian-faith-based-belief.html

B. Prokop said...

Ben,

You wrote: "So both Bob and William ... conflate physics with metaphysics" Why do you say this, when I wrote "Today we no longer take Aristotle to be the last word on How the World Works. This does not in the least invalidate Aquinas's logic"? How is that conflating the two, since you can see that I made a clear distinction between them?

Then you write, "they Ad Hoc claim Aristotle's metaphysics can't be separated from his physics" when, as you can see from the above and other things I wrote on this thread (such as "[Aquinas] needs to be re-expressed in a 21st Century framework. The reasoning does not have to be altered"), I most certainly separate them?

Just settin' the record straight here...

Son of Ya'Kov said...

Bob I took the following at face value.

"I will go out on a limb here and back up William's posting. Aquinas's argument as it stands is predicated on an acceptance of Aristotelian Physics."


Aquinas argument as it stands is not predicated on an acceptance of Aristotelian Physics but his metaphysics.

B. Prokop said...
This comment has been removed by the author.
B. Prokop said...

You're right. That sentence was badly written. It should have been "The presentation of Aquinas's argument as it stands is predicated on an acceptance of Aristotelian Physics." That is my major beef with it. It desperately needs a modern re-telling.

But, to repeat myself yet again, I don't put any great stock in "proofs" (other than ones for the historicity of the Resurrection). Even when they are spot on, I can't imagine anyone actually basing his faith on such. I much prefer the argument from immanence. To quote the great poet Gerard Manley Hopkins, "The world is charged with the grandeur of God".

(Which brings up a side point: I think that poetry, music, architecture, and the arts in general are far more effective apologetic tools than is prose. Name me even one great atheist work of art that can stand up to Eliot's Four Quartets, Beethoven's Missa Solemnis, the village church at Kilpeck, or Raphael's Alba Madonna. I have boundless respect for Aquinas, but I'll take any one of those over the Summa any day of the week.)

B. Prokop said...
This comment has been removed by the author.
B. Prokop said...

Ya know what? I'm in a good mood this morning, so I'll give "the opposition" a running start. Here are a few good examples of great atheist art (it actually does exist):

Vremya, Vpered! (Time, Forward!), novel by Valentin Kataev

The Lenin Mausoleum on Red Square, structure designed by Aleksej Shchusev

Ivan Groznyj (Ivan the Terrible), film by Sergej Eisenstein, music by Sergej Prokofiev

Vesna (Spring), painting by Arkadij Plastov

Takikh Zhenshchin ne byvalo i ne moglo byt' v staroe vremya (Such Women Could Not and Did Not Exist in the Old Days), poster by Mikhail Soloviev

V.I. Lenin na tribune (V.I. Lenin on the Tribune), painting by Aleksandr Gerasimov

All of the above unquestionably great works of art. But I dare anyone to take their underlying philosophy as something to be admired/emulated!

Son of Ya'Kov said...

Cheers brother Bob!:-)

Robert Oerter said...

The quantum mechanical counterexample seems to be a valid one.

An electron in an excited state has the potential to drop into a lower state, emitting a photon in the process. What is the cause that turns this potentiality into actuality? According to QM, there is no such cause. It is a spontaneous event without anything to cause it.

Can someone explain to me why this isn't a counterexample to Premise 1?

rank sophist said...

Oerter,

In Aristotelian terms, an "uncaused" event is the appearance of something from absolutely nothing. It isn't merely the lack of one thing obviously moving another thing: it's the lack even of any actual (in the act/potency sense) conditions for which the appearance of "random" particles (for example) are even a potentiality. But, as David Albert wrote in his NYT review of Krauss's A Universe from Nothing, the quantum fields that spawn said particles are far from being "nothing". In fact, under Aristotelian terms, they're actualities; and they have the "potential" to spawn particles.

This is an extremely stripped-down explanation, and it doesn't cover other important elements (such as essence and the "four causes") at play here. Regardless, I hope it clears things up a bit.

B. Prokop said...

Ditto what "rank sophist" said.

The cause of a random movement of an electron from one state to another is the set of underlying "laws" (Damn these anthropomorphisms!) that govern said movement. In any case, an electron is, from the perspective of QM, a very large and complex object. The actual mechanisms behind its movements are all played out on a much, much finer scale, down at the particle level. And at the very bottom of the scale, we come to a very weird world of what is basically pure information and no true matter (as we would recognize it) at all.

In the end, you will find the problem of the chain of causation does not ever go away. What seems uncaused at the electron level is very much caused at the Planck level.

rank sophist said...

Also, re-reading your post, it seems like you're thinking about this in the wrong way. For the follower of Aristotle, the question to ask when an excited electron spawns a photon is not necessarily "where is the third party that causes the photon to appear?" In Aristotle's reality, actual substances possess final causality ("end-directedness"). The spawning of photons is a potential end (not as in "termination"; "goal" is closer) within excited electrons.

Therefore, the questions to ask are rather,

1. What causes ("actualizes") the electrons;

2. What conditions or fluctuations cause (actualize) the electrons' excitation; and

3. What causes (actualizes) the cause of the electrons and/or their excitement?

And so on, ostensibly forever. However, the First Way states that, because an infinite regress of this or any other causal chain explains nothing, some pure, unsustained actuality is metaphysically necessary to explain the change of anything at any moment.

I'll ask the more knowledgeable Aristotelians here to forgive and correct any errors I made above.

Robert Oerter said...

rank, I'm not talking about an uncaused creation of the world from nothing, I'm just talking about a transition from one quantum state to another. Your first comment isn't even relevant to the example.

As for your second comment, "The spawning of photons is a potential end (not as in "termination"; "goal" is closer) within excited electrons." is precisely my point. This is only potential. It needs something actual to turn potentiality into actuality. In QM, nothing plays this role.

My question is not about what caused the electron to be excited in the first place, but what causes it to transition to a lower level. And specifically, what causes it to transition AT THAT TIME.

B., the underlying laws can't be the cause that's required under Premise 1, because they only describe potentialities, and Premise 1 requires something actual to realize the potential in order for any change to happen. The law of gravity doesn't cause the Earth to orbit the Sun, the Sun's actual gravity does. (The law of gravity is there even if the Sun were removed: would the Earth continue to orbit?)

Under the Humean view of laws, they are only descriptions of how actual things actually behave, so they have no causal efficacy. Under the immanentist view, laws just tell us about the properties inherent in actually existing things. So either way, the laws themselves can't be the cause in the sense required.

Furthermore, if you instead claim that the laws of QM are themselves actual, then you still have a problem, because the laws exist at all times, and so they don't explain why the electron will transition at that particular time.

"In any case, an electron is, from the perspective of QM, a very large and complex object."

This is just wrong. The electron IS the particle level - there's no lower level.

Son of Ya'Kov said...

Robert Oerter,

Primitive persons on a remote island might conclude by observation rain cloud cause rain even if they are ignorant of the mechanics of condensation.

The problem with Quantum systems is the moment you observe them you change them. There is nothing to rule out non-local Hidden Variables as an actualizing agent.

Of course for any even to be truly un-caused a potency to become actual without an actualizing agent you need to show the event happens without sufficient conditions and without necessity to vindicate Hume.

Of course Anscombe destroyed Hume by pointing out his arguments conflated imagination with intellective conception. I'll deal with this more later when I have the time.

BeingItself said...

"There is nothing to rule out non-local Hidden Variables as an actualizing agent."

How could you rule that out Ben? Try asking the dragon in your garage.

Son of Ya'Kov said...

BI it's you who believes in the dragon in your garage. I certainly don't.

If an invisible dragon existed it would be a "being alongside other beings" thus by definition it can't be God as defined in the classic sense.

You didn't read the paper or any of the relevant material. At least Robert Oerter knows enough to try to bring Hume into this argument as a potential defeater for Aquinas. You don't know any of this from a hole in your head.

BeingItself said...

Ben,

Answer the question.

B. Prokop said...

The big problem with trying to extrapolate too much (philosophically) from QM is our understanding of the processes involved is still in its infancy. In fact, it hasn't finished being born! We really need to wait a century or two before we can even begin to say authoritatively what is and what is not happening down at that level.

"The electron IS the particle level - there's no lower level." Whilst you are correct that there is no known substructure at this time to an electron, the mere existence of positrons as well as electrons is predicated upon there being some means of distinguishing between the two, i.e., some characteristic substructure or property that determines the difference between the two. In other words, until you get to the level where there are no particles at all, but rather you find yourself in a realm of pure information, it's turtles all the way down.

Now this is just me speaking here, but as I said a couple of months back, in a universe of ultimately elemental particles, you have the unsolvable problem of explaining where the information is stored that "tells" said particles how to behave. Thanks to some (headache inducing) tutoring I've recently been exposed to, I now regard such a state of affairs as non-existent. It appears that at the ultimately fundamental levels of reality, what we call "matter" doesn't even exist. It's all information.

Son of Ya'Kov said...

>what causes it to transition to a lower level. And specifically, what causes it to transition AT THAT TIME.

As Anscombe points out Hume's weak argument is that he can with ease "conceive" of a ball coming into existence on his table from nothing uncaused by simply imagining it & that somehow disproves causality.

The thing is imagining a ball suddenly appearing on your table and merely labeling it "from nothing" isn't the same as conceiving of Nothing producing a ball as if nothing where something with the potential to produce a ball. Plus you can't even say you can imagine a ball coming from nothing since merely imagining a ball appearing "from nothing" is no different then imagining a ball being telephoned from somewhere & appearing before you.

The sudden appearance of ball is not proof of the un-caused actuality of the potential for the existence of a ball.

The unknown mechanics of how an electron transitions to a lower level. And specifically, what causes it to transition AT THAT TIME does not disprove causality.

Hume was wrong.

BeingItself said...

Again.

"There is nothing to rule out non-local Hidden Variables as an actualizing agent."

How could you rule that out Ben?

Shackleman said...

A relevant quote from Berkeley:

"It is evident to any one who takes a survey of the objects of human knowledge, that they are either ideas actually imprinted on the senses, or else such as are perceived by attending to the passions and operations of the mind, or lastly ideas formed by help of memory and imagination, either compounding, dividing, or barely representing those originally perceived in the aforesaid ways. By sight I have the ideas of light and colours with their several degrees and variations. By touch I perceive, for example, hard and soft, heat and cold, motion and resistance, and of all these more and less either as to quantity or degree. Smelling furnishes me with odours; the palate with tastes, and hearing conveys sounds to the mind in all their variety of tone and composition. And as several of these are observed to accompany each other, they come to be marked by one name, and so to be reputed as one thing. Thus, for example, a certain colour, taste, smell, figure and consistence having been observed to go together, are accounted one distinct thing, signified by the name apple. Other collections of ideas constitute a stone, a tree, a book, and the like sensible things; which, as they are pleasing or disagreeable, excite the passions of love, hatred, joy, grief, and so forth.
As this passage illustrates, Berkeley does not deny the existence of ordinary objects such as stones, trees, books, and apples. On the contrary, as was indicated above, he holds that only an immaterialist account of such objects can avoid skepticism about their existence and nature. What such objects turn out to be, on his account, are bundles or collections of ideas. An apple is a combination of visual ideas (including the sensible qualities of color and visual shape), tangible ideas, ideas of taste, smell, etc.[12] The question of what does the combining is a philosophically interesting one which Berkeley does not address in detail. He does make clear that there are two sides to the process of bundling ideas into objects: (1) co-occurrence, an objective fact about what sorts of ideas tend to accompany each other in our experience, and (2) something we do when we decide to single out a set of co-occurring ideas and refer to it with a certain name (NTV 109).


Thus, although there is no material world for Berkeley, there is a physical world, a world of ordinary objects. This world is mind-dependent, for it is composed of ideas, whose existence consists in being perceived. For ideas, and so for the physical world, esse est percipi."

http://plato.stanford.edu/entries/berkeley/#3.1

Son of Ya'Kov said...

>Ben,

>Answer the question.

Why you refuse to answer any of mine.

F*** off!

BeingItself said...

Coward.

You do not even understand my "Dragon in the Garage" reference.

You continually make the same moves the Dragon believer does. It's hilarious. You really need to read the chapter.

Son of Ya'Kov said...

Whatever BeingItself!

Bye!

BeingItself said...

Ben's logic:

If my explanation cannot be ruled out, then my explanation is True.

Son of Ya'Kov said...

BeingItself

I'm not arguing with you. You are not arguing in good faith. You are not arguing at all just using a set of classic troll moves.

http://www.teamtechnology.co.uk/troll-tactics.html

Give it a rest.

William said...

BI: Carl Sagan's dragon in the garage was a way of dealing with claims of facts which had no empirical support. Ben's assertion is almost the opposite: that empirical facts exist, but that a cause for them must exist though we cannot see this directly. This is in fact something Sagan might have agreed with.

My problem is that Ben seems to defend the First Way's empirical dis-confirmation by transmuting it into a Fifth Way argument, but it remains that it FAILS as it stands (g&d)

Son of Ya'Kov said...

William you have got scientism on the brain.

>My problem is that Ben seems to defend the First Way's empirical dis-confirmation by transmuting it into a Fifth Way argument, but it remains that it FAILS as it stands (g&d)

William if all you think you know about the Five ways comes from my incomplete comments here then you are worst than BI.

Here do the back-round reading.

http://edwardfeser.blogspot.com/search?q=ID+roundup

You should learn the difference between the 5th way vs Paley's bullshit.

http://edwardfeser.blogspot.com/2009/09/teleology-revisited.html

Enough of the ready fire aim already!!!

Son of Ya'Kov said...

You may want to check this one out too William.

So you think you understand the Cosmological Argument?

http://edwardfeser.blogspot.com/2011/07/so-you-think-you-understand.html

rank sophist said...

rank, I'm not talking about an uncaused creation of the world from nothing, I'm just talking about a transition from one quantum state to another. Your first comment isn't even relevant to the example.

I wasn't talking about the creation of the world from nothing, either. I was speaking in Aristotelian terms about causation. In that sense, it's as absurd to suggest that a rubber ball could come from "absolutely nothing" (as in, it had no cause in any way) as it is to say that the universe could come from "absolutely nothing"--there is no difference on a metaphysical level. In any case, Aristotle believed that the universe was eternal, so it's doubly irrelevant.

As for your second comment, "The spawning of photons is a potential end (not as in "termination"; "goal" is closer) within excited electrons." is precisely my point. This is only potential. It needs something actual to turn potentiality into actuality. In QM, nothing plays this role.

I'm no student of QM, so I'll have to ask you to bear with me. My previous posts were confused with regard to the subject; please ignore what I said before.

It sounds as though, like B. Prokop said, the reason hasn't been discovered but probably could be discovered. Therefore, suggesting that this QM example disproves act/potency sounds to me like, to paraphrase Feser, a "skepticism of the gaps" argument. In other words, it's a variation of kind of flawed proposition that ID theorists are accused so regularly of attempting.

My question is not about what caused the electron to be excited in the first place, but what causes it to transition to a lower level. And specifically, what causes it to transition AT THAT TIME.

Like I said, I'm no QM expert. And I don't think you're going to find a scientific breakthrough like that in a combox. The point here is that this holds no more weight as an argument than does an ID claim about the "irreducible complexity" of the eye. It merely illustrates the limits of today's scientific knowledge. Proposing that we toss out act/potency (in your case) or evolution-sans-interference (in ID's case) as a result of these limitations is just silliness.

The location of flaws in act/potency requires metaphysical demonstration of the kind Kenny attempted in the '60s, which Oderberg was seeking to refute in the article Mr. Reppert posted. Anything less is guaranteed to be inadequate.

BeingItself said...

William, you are wrong. Ben is using the exact same kind of idiotic arguments the Dragon in the Garage believer does.

For example, Ben says "There is nothing to rule out non-local Hidden Variables as an actualizing agent."

When asked what sort of evidence could "rule out" such a thing, Ben is silent. Likewise, the Dragon believer is silent about what kind of evidence would show there was no Dragon in the garage.

Ben, like the Dragon Believer, holds his belief dogmatically. He will use special pleading to hold onto it come what may.

It's is useless trying to talk with such a person.

Son of Ya'Kov said...

@BeingItself

Whatever you say BI. Your a deep thinker.

BeingItself said...

"Your a deep thinker"

You're a deep thinker.

Fixed it for ya. Genius.

Son of Ya'Kov said...

Well I is infamous for my bad grannma and speklling.

I own it.

Of course I don't pretend to have read philosophical papers I clearly haven't unlike some people I know.

William said...

Is it really the case that both BeingItself and BenY seem to think that whether or not God exists can be tied to a particular alternative interpretation on QM, but they differ on which interpretation of QM is correct?

If so, we have the potential for an eventual empirical test for God. Fascinating :).

BeingItself said...

No.

I am open to the possibility that some events occur uncaused.

But Ben must dogmatically deny this possibility because it undermines his 'proof'. So he makes idiotic statements like "There is nothing to rule out non-local Hidden Variables as an actualizing agent."

Of course it is impossible to "rule out" magic as a cause as well. Or undetectable quantum elves.

Son of Ya'Kov said...

>Is it really the case that both BeingItself and BenY seem to think that whether or not God exists can be tied to a particular alternative interpretation on QM, but they differ on which interpretation of QM is correct?

No it's not.

William I don’t believe at all in empirical arguments for the existence of God. I believe the existence of God can only be proved philosophically. I have repeatedly said so & for some asinine reason you have chosen to ignore my words.

BI is a troll who has not taken this topic seriously from the beginning nor contributed anything worth while.

Robert Oerter has tried to make the case from Quantum physics as interpreted by Hume’s metaphysics and philosophy that a potency can somehow actualize itself.

Which kind of begs the question since all he needs to do is argue from Hume against the classic metaphysics. There are Aristotelian interpretations of Quantum Mechanics by physicist philosophers like Ian Thompson and others.

http://www.u.arizona.edu/~aversa/scholastic/

>If so, we have the potential for an eventual empirical test for God. Fascinating :)

You have so not been paying attention & I do believe you have scientism on the brain.

Son of Ya'Kov said...

BTW in defense of Science against the Gnus!

The Non-local hidden-variable theory of Quantum Mechanics is part of historic Quantum Physics and was championed by David Bohm. Bell's theorem eliminated the idea of local hidden-variables but not non-local.

I find it very entertaining what an ignorant faker BI is & like most unlearned Gnu Atheist types he is a science fetishist not someone who has even a passing familiarity with any real science that he claims to believe in over and against God.

"undetectable quantum elves." comical!

I can respect even if I disagree, the Atheism of philosophers and learned scientists who know and respect philosophy.

But the childish ignorant rants of philistines not so much.

Crude said...

Wow, I see Ben's been going at it.

I'll simply weigh in with this comment. I'm surprised no one else seems to have made it yet.

We have no observation of something coming into existence from utter nothingness without cause. Period. What's more, it's impossible in principle to observe this - the absolute best you can ever hope for is to observe something that seems to come into existence, and you are unable to identify the cause.

Period. End of discussion. Observing utter nothingness, and observing the lack of a cause, isn't available.

Now, you can always throw up your hands at any given point and say "Maybe it came into existence from utter nothingness and without cause!" Notice how this is ridiculously similar to 'God did it'. It's not a conclusion of science, certainly not empirical research. It's an abdication of both.

Bohm isn't required here, except to illustrate that there are multiple interpretations of quantum physics. I hope you guys realize that said interpretations aren't themselves scientific, and in the most ultimate (and here, appropriate) sense, they have no hope to be so.

BI could have saved everyone a lot of time and virtual ink by simply saying "I'll sooner sacrifice science and reason rather than follow any chain of logic which concludes in God existing". He certainly doesn't need quantum physics as his example - he can do it for anything.

Why does Mars exist? Hey, maybe it popped into existence from utter nothingness, uncauased. Would Mars' origin having a cause prove God's existence? Then it damn well didn't have a cause. ;)

rank sophist said...

I'll simply weigh in with this comment. I'm surprised no one else seems to have made it yet.

We have no observation of something coming into existence from utter nothingness without cause. Period. What's more, it's impossible in principle to observe this - the absolute best you can ever hope for is to observe something that seems to come into existence, and you are unable to identify the cause.

Period. End of discussion. Observing utter nothingness, and observing the lack of a cause, isn't available.

Now, you can always throw up your hands at any given point and say "Maybe it came into existence from utter nothingness and without cause!" Notice how this is ridiculously similar to 'God did it'. It's not a conclusion of science, certainly not empirical research. It's an abdication of both.


My third reply to Oerter was an attempt to get some of this across, but you expressed it more clearly. Props.

Crude said...

rank,

I saw your comments and thought you may well be saying something similar, but I wasn't totally sure. Glad to see we're on the same page either way.

Robert Oerter said...

There's an awful lot of special pleading going on here. If I say, "Uniform motion is a change without an efficient cause," I'm told that uniform motion doesn't count. If I say "Quantum transitions are changes without an efficient cause," I'm told there could be a cause we don't know about.

Sorry, but that doesn't cut it. The question is not whether I can rule out hidden variables, but whether you can rule them in. The premise is, "Everything that is changing is being changed by something else." Unless you can exhibit the "something else" that is causing the quantum transition, QM stands as a counterexample. You can't change the premise to, "Everything that is changing is being changed by something else, and if we can't see a cause we'll assume there's one anyway."

QM has been subjected to increasingly stringent tests for almost 100 years and there has never been any evidence for hidden variables. If you succeed in demonstrating their existence, there's a Nobel with your name on it, so go to!

grodrigues said...

@Robert Oerter:

" If I say, "Uniform motion is a change without an efficient cause," I'm told that uniform motion doesn't count."

Are you appealing to the first Newton's law of motion? That law that is not empirically falsifiable because we cannot observe, not even in principle, free bodies with no forces acting on them? That law that presumes an absolute, homogeneous and isotropic Galilean space-time with a special class of coordinate frames (inertial frames) and that has been superceded by General Relativity? Maybe you should read the following, before jumping into facile conclusions about the validity of Quidquid Movetur ab Alio Movetur (link between quotes because there are spaces in there):

"http://www.thomist.org/jourl/1974, vol. 38/April/1974 April A Moreno web.htm"

"The premise is, "Everything that is changing is being changed by something else." Unless you can exhibit the "something else" that is causing the quantum transition, QM stands as a counterexample."

First, no, QM is not a counterexample. You are reading your philosophical (mis)interpretation into the abstract formalisms of quantum mechanics. If you *really* want to argue that QM is a counterexample to such a basic metaphysical principle, hand-waving your arms about is not enough, construct your argument. As first steps you have to:

1. Justify the complete absurdity that is saying that there are uncaused events. Science *is* knowledge via causal explanations, so what you are saying is the death sentence for all science -- I do not think you really appreciate this point, so forcefully put by Crude.

2. If you want to support the idea that events at the (sub)atomic level are uncaused than you have to explain exactly what you mean by cause writ large. Since this is not a question of physics, QM will not help you in the least. You are already dead in the water here, because most probably all you have is a shriveled carcass of a notion of efficient causality that Thomists will reject anyway.

3. Related to 1. but slightly different, QM is an abstract mathematical formalism whose job is, similar to every other physical theory, to correctly predict the correlations measured in experiments. Making ontological statements from theoretical constructs is to confuse the map for the territory. Argumentation is needed, and since the matter is at bottom, a metaphysical one, QM alone cannot do the job for you -- although it can certainly inform the arguments.

4. Since there are fully causal and deterministic interpretations of QM you have to justify your particular pick (most probably, Copenhagen). Since this cannot be decided empirically, at least not yet, physics will not help you.

Second, you are turning a gap in our knowledge (there is much we do not understand about what happens what the subatomic level, and for all we know, we may never fully understand it) into a definite metaphysical statement -- I will dub this the atheism-of-the-gaps argument. So the burden of proof in on you to construct the argument: what seems to you to be absent is really absent. And then deal with the consequences -- they ain't pretty as adumbrated in bullet 1.

BeingItself said...

Robert Oerter's last comment was spot on. No matter what counter examples are shown to any premise for these idiotic "proofs", you guys will always special plead your way out of it.

Robert says:

You can't change the premise to, "Everything that is changing is being changed by something else, and if we can't see a cause we'll assume there's one anyway."


But Ben thinks he can. He does not use the rules of reasoning that rational people use. Special pleading and begging the question are his only tools. Just like the guy who is sure he has a dragon in his garage.

Son of Ya'Kov said...

BI there is nothing wrong with being a cheerleader for your side. I've done it myself but as long as you realize that's the upper limit of your skills. That and good spelling and grammar.

Crude said...

Robert,

If I say "Quantum transitions are changes without an efficient cause," I'm told there could be a cause we don't know about.

No, Rob. What you're being told is that your claim is not backed up by science or established by empirical data - and that it can never be so, even by some future science. Because 'this thing came into existence from utter nothingness, without any cause at all' simply isn't an option available to empirical confirmation.

I'm not arguing with the findings of quantum physics, because said findings - insofar as they are a science - simply do not get to the conclusion you're aiming for here.

I know, I know: you've been assured that quantum physics is quite weird. (It is!) You've been assured that quantum physics is very counterintuitive. (It is!) But that didn't make the nonsense in What The Bleep Do We Know true - and it doesn't make your move here true.

You can't change the premise to, "Everything that is changing is being changed by something else, and if we can't see a cause we'll assume there's one anyway."

No premise is being changed, and the requirement has never been 'We have to see/know the cause!' C'mon man, think of when these arguments were first formulated. Do you really think they were made with the complete knowledge of all causes ready and demonstrable? Sorry, you've been misinformed.

And as I said with BI, you don't need quantum physics here. Point at anything you wish, from Mars to otherwise. Insist, "I think it could have popped into existence from utter nothingness without a cause! You can't prove otherwise beyond a shadow of a doubt!" What you're doing is sacrificing science and empirical knowledge. And hey, you're welcome to it!

Just don't pretend it's science.

Crude said...

BI,

He does not use the rules of reasoning that rational people use. Special pleading and begging the question are his only tools.

Bwahaha. Yeah, BI. Ben isn't willing to use appeals to magic and irrationality as tools - poor him. Yes, I know, usually the cultists of Gnu like to insist that everyone else is dabbling in magic and irrationality, but look at you. You're sitting there saying 'Some things happen or come into existence utterly without cause from total nothingness! That's my objection!' You're welcome to it.

Just as the YEC is welcome to make the move of 'Well, Mister Evolution-man, you haven't answered my master argument yet: maybe the world popped into existence 6000 years ago looking exactly as it does! Game, set, match!' In fact, they're damn similar moves. ;)

BeingItself said...

"Some things happen or come into existence utterly without cause from total nothingness!"

I never said that. So lying is the main tool in your toolkit.

Crude said...

"Some things happen or come into existence utterly without cause from total nothingness!"

I never said that. So lying is the main tool in your toolkit.


Bull.

Earlier: "I am open to the possibility that some events occur uncaused."

So what's your dodge going to be? "I didn't say it happens - I just am open to the possibility!"?

Go ahead - make your move.

rank sophist said...

Sorry, but that doesn't cut it. The question is not whether I can rule out hidden variables, but whether you can rule them in. The premise is, "Everything that is changing is being changed by something else." Unless you can exhibit the "something else" that is causing the quantum transition, QM stands as a counterexample. You can't change the premise to, "Everything that is changing is being changed by something else, and if we can't see a cause we'll assume there's one anyway."

QM has been subjected to increasingly stringent tests for almost 100 years and there has never been any evidence for hidden variables. If you succeed in demonstrating their existence, there's a Nobel with your name on it, so go to!


There isn't much I can say that hasn't been said already, but let me just go back to my evolution example to illustrate the move you're making.

1. Finding E is currently unexplained according to accepted structure S.

2. S is a foundational concept in our system, and tossing it out completely would destroy most or a large portion of human knowledge.

3. Nothing inherent in E prevents it from being explained according to S once better tools are available.

Replace "E" with your QM example, or with "irreducibly complex" evolutionary phenomena, or with anything else appropriate. Replace "S" with act/potency, undirected evolution or, again, anything else appropriate.

The decision to toss S would never be made under these circumstances. Such a decision would need to be based on bigger issues in S, which are most likely independent of E. In act/potency's case, these would be high-level metaphysical errors; in evolution's case, these would be high-level theoretical errors. Which once again brings me to my original point: to disprove act/potency, you have to take Kenny's route of dismantling it at its core. Your current method is inadequate.

Also, the "dragon in the garage" accusation does not apply here. True, E could be taken as "the dragon's apparent non-existence when examining the garage", and S could be taken as "the understanding that the dragon is in the garage". However, in this case, tossing S would not "destroy most or a large portion of human knowledge".

Robert Oerter said...

OK, guys, I'm confused. Are you saying that Premise 1 is a metaphysical principle that is true a priori, or that it is an empirical principle?

If it is the former, then you need to demonstrate an a priori argument for it. Maybe you think this has been done elsewhere - but your comments above don't even attempt to make such an argument.

If you think it is the latter, then you have to accept the empirical evidence of QM as part of what needs to be shown to be consistent with the premise. Oderburg at least engages with the issue of uniform motion, and attempts to make a philosophical argument for why it is not a counterexample. (I'm not convinced he succeeds, but let's let that pass for now.)

What you have utterly failed to do is make any sort of similar argument for QM. To insist, as Crude does, that the "claim is not backed up by science or established by empirical data - and that it can never be so, even by some future science," is to abandon any attempt to explain how QM is consistent with the premise.


You are, of course, free to simply assume Premise 1 as an axiom. (Or you could go ahead and simply assume God as an axiom and avoid the need for any sort of argument in favor of God.) But you can't expect anyone to find the First Way a convincing argument if you just declare the premises true by assumption.

Robert Oerter said...

grodriguez, thanks for the link - it was very interesting and helpful. Moreno concludes:

"And these considerations should also be taken into account in order to evaluate the validity of Quidquid movetur ab alio movetur. The principle appears as an immediate consequence of the definition of motion. But since in the case of local motion the true causes of it are unknown, the specific interpretation of the principle is also unknown, because this interpretation depends upon these unknown causes."

If I understand him correctly, his position is (1) Premise 1 is an a priori truth, and (2) we really don't understand inertia, so we don't know how Premise 1 applies in the case of local motion.

Is this your position with respect to QM, too?

rank sophist said...

Oerter,

The necessity of the act/potency distinction has been clear since the Greeks. Oderberg himself makes a very long argument to establish its importance in his book Real Essentialism. No one is saying that it's an a priori truth. The standard logical axioms are of course required if one wants to understand it, but I don't think anyone wants to go through a defense of those in a combox. I'm sure you can understand that rehashing more than two thousand years worth of argument about them would waste everyone's time, including your own.

If you want to know what a world without the act/potency distinction would look like, see Parmenides and a few of the other ancient Greek philosophers.

If you think it is the latter, then you have to accept the empirical evidence of QM as part of what needs to be shown to be consistent with the premise.

Did you read what I wrote in the post above yours? I apologize if it wasn't the most intuitive. In any case, I believe I showed that this move does not work. It's a disguised version of the intelligent design argument.

grodrigues said...

@Robert Oerter:

"If I understand him correctly, his position is (1) Premise 1 is an a priori truth, and (2) we really don't understand inertia, so we don't know how Premise 1 applies in the case of local motion.

Is this your position with respect to QM, too?"

In the part you quoted, Moreno explicitly says that the principle follows from the definition of motion (e.g. change). And might I add, from the primary division in being, that between potency and act. I am with rank sophist here (great nick); it is not an a priori truth (really lousy terminology), but a metaphysical first principle. We can justify it in two ways: give positive reasons that compel us to accept it and show that its denial leads to absurdities. Since it is a first principle of *being*, such considerations will of necessity appeal to the nature of reality, the *whole* of reality not just the tiny corner of it accessible to the modern empirical sciences. Maybe I am misreading your (2) but for the sake of clarity, it is not that premise 1 does not apply to the case of local motion; of course it does. What we do not know is how to apply it in this specific case because we lack the full causal explanation for inertia -- in such a condition of ignorance, it is a tad difficult to identify the alio movetur.

And by the way, did you miss Moreno's discussion of Aquinas' interpretation of the principle as contrasted with Averroes? If he is right, and I think he is, then we do not even have to invoke lack of knowledge because it then becomes clear that local uniform motion does *not* violate the principle (I think Ben Yachov said something similar to what Moreno is saying, but I would have to go back in the thread to be sure). This is important because it highlights other points where your objections goes wrong.

1. First, local uniform motion is just one case of motion (change). Even if I were to grant your interpretation of it, it would still be insufficient to undermine Aquinas' argument, because in order for Aquinas' argument to go through, there is no need for the principle to apply to *every* case of change, *one* is enough. This puts a considerable burden on those who want to refute the argument -- good luck with that.

2. The explanation of Newton's first law itself needs an explanation. It is not enough to just invoke it, for what we want to know is *why* exactly are material bodies governed by such a law. Aquinas' would respond, in the modern terminology, that the laws of Nature are the laws of natures which of course invites the question of why material bodies have just such a nature and this leads once again to the Unmoved Mover.

With a preemptive caution against conflating the two distinct levels of explanation, the metaphysical and the physical one, the above pretty much applies verbatim to the case of QM, thus answering your question. As I said in my previous post, and as other people have also said it, QM in and of itself does not, and can never, suffice to undermine the principle. If you want to combat what you deem as bad metaphysics, then you must do it on the metaphysical battlefield.

Son of Ya'Kov said...

>Are you saying that Premise 1 is a metaphysical principle that is true a priori, or that it is an empirical principle?

Thesis 22

"We do not perceive by an immediate intuition that God exists, nor do we prove it a priori. But we do prove it a posteriori, i.e., from the things that have been created, following an argument from the effects to the cause: namely, from things which are moved and cannot be the adequate source of their motion, to a first unmoved mover; from the production of the things in this world by causes subordinated to one another, to a first uncaused cause; from corruptible things which equally might be or not be, to an absolutely necessary being; from things which more or less are, live, and understand, according to degrees of being, living and understanding, to that which is maximally understanding, maximally living and maximally a being; finally, from the order of all things, to a separated intellect which has ordered and organized things, and directs them to their end."

For what this is worth thought Thomistic a posteriori investigation is not the same as Hume's empiricism. Hume would travel the Cosmos searching for God liek in Star Trek 5. Aquinas would start with a rock and never leave the Monastery.

Robert Oerter said...

Thanks for the responses, guys. I think this goes a long way to explaining our difference of opinion about the burden of proof. I was thinking of Premise 1 as an empirical claim. Maybe because my physicist brain reads it automatically as such.

I don't make any claims to philosophical sophistication, and of course I don't expect you to clear up all my confusions in this thread. I do appreciate the suggestions for reading, and would like to hear what else you recommend. (This is the second time I've been told to read Parmenides in the last few weeks.) Metaphysics is probably my weakest area.

grod,thanks for the comments. I would like to see what this looks like: "We can justify it in two ways: give positive reasons that compel us to accept it and show that its denial leads to absurdities."

"...there is no need for the principle to apply to *every* case of change, *one* is enough."

Surely this is wrong? After all, the principle in question states
"EVERYTHING that is changing is being changed by something else." And how can we argue to a first mover if it doesn't have to apply to every case?

But maybe I'm misunderstanding you.

Son of Ya'Kov said...

Robert Oerter,

Two types of Atheists I respect. Atheist philosophers and those who don't know philosophy but admit they need to learn.

(Gnus of course I hold in infinite contempt like any other mindless fundie)

You seem to be the second type of non-Gnu.

Props!

I recommend THE LAST SUPERSTITION not so much to convince you of the possible existence of God but it will give you a good overview of the break between Classic vs Modern philosophy and it will show you how philosophical inquirery is just as valid a form of natural knowledge as empirical science.

Cheers.

grodrigues said...

@Robert Oerter:

"grod,thanks for the comments. I would like to see what this looks like: "We can justify it in two ways: give positive reasons that compel us to accept it and show that its denial leads to absurdities.""

Ben Yachov already gave you one reference. I should warn you though that the book has a fairly polemical tone; I love polemical but some supercilious people seem to be put off by it. An online account of the Thomistic synthesis can be found in (jump to chapter 4),

http://www.thesumma.info/reality/index.php

but it is harder to digest than Prof. Feser's book which is fairly easy going (buy I am a mathematician so my judgment may mean little, as I eat abstractions for breakfast).

"Surely this is wrong? After all, the principle in question states "EVERYTHING that is changing is being changed by something else." And how can we argue to a first mover if it doesn't have to apply to every case?"

What does the argument purport to establish? The existence of a first unmoved mover. There are two important qualifiers here: first and unmoved. First here does not have to be construed as first mover for *all* chains of causation; after all those who believe on Free Will will say that humans are agents and can originate chains of causation (although a posteriori it really does happen that Unmoved Mover is first in a universal sense even when Free Will is at stake). As long as one particular chain of causation satisfies the principle -- which I insist, does follow from the definition of change and other metaphysical first principles such as the division between potency and act -- and you can establish the existence of a first unmoved mover you are basically all set. Of course, it is indisputable that if the principle applies universally to all chains of causation, it is much easier to make the argument stick. And one could argue that establishing the principle for one chain of causation is about as difficult as establishing it for all chains of causation, so we are no better off. Without having thought too much about it, this does seem reasonable, but it also puts a stronger burden on those wishing to refute the argument because it means that they have to say that the principle fails for *every* chain of causation, which is tantamount to saying that every chain of causation ends up, tracing it backwards in the direction of the source, in a brute fact, which is tantamount to saying that no chain of causation is really explainable or intelligible.

William said...

RO: "I was thinking of Premise 1 as an empirical claim."

As interpreted here, it seems that Premise 1 is exactly as empirical a claim as is stating that a deterministic variety of QM is the true interpretaion of QM.

BeingItself said...

I too recommend The Last Superstition. It's fun to read, and gives a good lesson in philosophy done badly. A physicist in particular will find Feser's gaff's adorable. There are some hilarious mathematical blunders as well.

grodrigues said...
This comment has been removed by the author.
grodrigues said...

@BeingItself:

"A physicist in particular will find Feser's gaff's adorable. There are some hilarious mathematical blunders as well."

What are these famous physical and mathematical blunders? I have found none, so I am curious to know what I have missed.

Son of Ya'Kov said...
This comment has been removed by the author.
Son of Ya'Kov said...

This should be entertaining.

grodrigues has forgotten more math theory then anyone here has learned.

BI is a wannbe faker.

Son of Ya'Kov said...

>I too recommend The Last Superstition. It's fun to read, and gives a good lesson in philosophy done badly.

Of course if memory serves BI was bashing philosophy in typical Gnu fashion and obstinately refused to read anything.

I doubt he's read Feser.

If he has then what's his excuse for claiming the First Way is an argument from Aristotle's folk physics since Feser shows meticulously it was a response to the metaphysics of Parmenides?

Or maybe he is just a troll who contradicts for the sake of contradicting. He has no worthwhile arguments.

At least Robert Oerter brought up Hume. He takes this seriously.

BI doesn't.

BeingItself said...

Feser tries to make a distinction between an essentially ordered causal series and an accidentally ordered causal series. He then gives several examples of what he thinks are essentially ordered but they are all accidentally ordered. It's quite funny.

(For the record, I don't think "essentially ordered causal series" exist in nature.)

Those few pages were really embarrassing for Feser. Also, it exposes that most of this "metaphysics" you guys go on about is just made up non-sense.

Crude said...

Mmm, gotta love that vagueness on BI's part. C'mon, BI - give the quotes and page numbers. Make your case.

Of course, even if Feser gave a mistaken example, it wouldn't harm his case. But let's see if he did even that. Especially the 'mathematical blunders'.

Son of Ya'Kov said...

BI has never read the book he's a pathological liar. That's while he won't produce page numbers.

Son of Ya'Kov said...

Of course the "example" I think he is giving isn't even his own original objection. It was made by a nutjob Gnu named unbeguiled.

Simply copying some other Gnu's unsubstantiated and long refuted charge of error is lazy not to mention plagiarism.

BI likely got it from Feser's blog or did a google search and got it from Unbeguiled. He certainly didn't read the book anymore then he read Oderberg's paper.

Papalinton said...

BeingItself
"Those few pages were really embarrassing for Feser. Also, it exposes that most of this "metaphysics" you guys go on about is just made up non-sense."

Of course it is all made-up stuff. Catholics have been making up stuff since time immemorial. Where do you think they get their expertise from? The recent greatest joke is the notion of Papal infallibility, formalized as a gift from god in the mid-1800s. How is that for pious fraud and god-made-me-do-it nonsense.

Limbo was another inviolable message that was in because god said so, then it was out, then in again. Can one even imagine the anguish, the pain, the emotional agony and despair of those mothers, whose children died before baptism, at a time when 'limbo' was dogma, knowing they could never be reunited with their dead child, simply because some moral monster masquerading as a Pope, and in glorious depraved indifference, told those grieving mothers their children are lost in limbo for eternity.

And now, once again out of favour, all those countless infants that died before baptism are now understood to not be consigned to limbo for eternity.

Religion is all made-up nonsense.

grodrigues said...

@BeingItself:

Still waiting for the famous mathematical and physical blunders that should give me a fit of laughter. With references to the pages please, so I can check them up.

And I should note that Feser is a philosopher and contrary to other ignorant loudmouths that will go unnamed, the only competence he claims to have is in philosophy, and even then only in some of its fields. I doubt that even if there were indeed some mistakes, they would harm his case.

Anyway, I am indeed curious. Laughing is good.

Robert Oerter said...

grod, I appreciate the point about only needing one causal chain. But then you're talking about a very different premise than our Premise 1. As it stands, there only need be one exception to disprove Premise 1.

If QM is indeed the correct view of physical reality at its most basic level, then ALL causal chains are broken by spontaneous quantum jumps of the kind I mentioned above.

I read Feser's blog from time to time. I find him annoying but often right, so I'll probably be able to stomach the polemics. I've been avoiding TLS, but maybe I need to put it on my reading list. Sigh.

Son of Ya'Kov said...

>grod, I appreciate the point about only needing one causal chain. But then you're talking about a very different premise than our Premise 1. As it stands, there only need be one exception to disprove Premise 1.

The problem with trying to find your Atheist causal Gap is that it's fundamentally irrational.

It's like saying "All I need is to find one instance of 2+2=5 to empirically invalidate 2+2=4."

Even in some hypothetical Universe which has some exotic physical laws where whenever you take a set of 2 objects and put it with another set of 2 objects the Quantum Vacuum spontaneously generates a fifth object in such a realm 2+2 would still equal 4 & you would just have a case of (2+2)+1=5 not 2+2=5.

Still I wish you well in reading Feser. Ignore his politics. I enjoyed Eric Retain's writings even thought he was to the far left of my politics.

Learn philosophy even if you don't believe in God you will be at least a better Atheist for it.

grodrigues said...

@Robert Oerter:

"If QM is indeed the correct view of physical reality at its most basic level, then ALL causal chains are broken by spontaneous quantum jumps of the kind I mentioned above."

Sorry for nitpicking (professional deformation, I guess) but you must replace "QM" by "my metaphysical interpretation of the abstract mathematical formalisms of QM" to get a true statement -- for the reasons already mentioned in previous posts and that I will not repeat here.

William said...

grodrigues:

"replace "QM" by "my metaphysical interpretation of the abstract mathematical formalisms of QM
"

---

The First way's philosophical logic, as a proceeding from definitions and axioms to its result, is valid. The problem with all such arguments are the need for premises that may not really be obvious when examined in detail.

QM is about the actual data produced by real equipment measuring real physical things, AND the arguments of Aquinas are about the interactions of an actual prime mover with real things. The associated formalisms are not supposed to be _just_ about abstract mathematical reasoning!

We ought to be sure our required premises are at least as EMPIRICALLY acceptable as our conclusions. Which a required selection of the less popular deterministic interpretation of QM is not (Einstein's initial views excepted, I'll admit).

The First Way is supposed to reason from obvious axioms to its inevitable conclusion. Is _your_ required interpretation of QM really that obvious to you? I personally object to it because I dislike determinism.

grodrigues said...

@William:

"The problem with all such arguments are the need for premises that may not really be obvious when examined in detail."

If there is a problem it surely is not that one, it is that the premises are not true.

"The associated formalisms are not supposed to be _just_ about abstract mathematical reasoning!"

Agreed, but I never said that anyway. What I said was that you cannot simply read off metaphysical conclusions from the abstract formalisms of QM whose job is to accurately describe and predict the correlations measured in experiments. Physical theories do not come with a preferred metaphysical interpretation attached so argumentation, as opposed to hand-waving, is needed.

"We ought to be sure our required premises are at least as EMPIRICALLY acceptable as our conclusions. Which a required selection of the less popular deterministic interpretation of QM is not (Einstein's initial views excepted, I'll admit)."

There is no empirical way to differentiate the interpretations, so I do not understand what you are trying to say.

"Is _your_ required interpretation of QM really that obvious to you? I personally object to it because I dislike determinism."

I did not know I had a "required interpretation of QM". You seem to think that my metaphysical commitment to AT forces me to choose a deterministic interpretation, but that is simply not true. Or to put it more precisely, it depends on the brand of indeterminism you are trying to sell me. I do reject all the nonsensical, un-scientific tosh about "uncaused events" and I most definitely reject Copenhagen for various philosophical reasons (and the debate *is* by and large philosophical) that I will not go over here. The fact is that I have no settled opinion on the matter but I do tend to sympathize with determinist, non-local interpretations. I won't explain the reasons, suffice to say that I am fully aware that non-locality opens up a very nasty can of worms.

note: there is a promising program by C. Isham and collaborators to restore local determinism to QM, but only time and more work will tell if it pans out. Actually their aim is Quantum Gravity, but since say, Copenhagen, poses vast problems for QG they are forced to tackle this issue.

I should add as a final note, and actually the most important note, that you keep reducing everything to empirical considerations, when the important battle lines are drawn in the metaphysical plane. QM alone and by itself, qua scientific theory, will *not* do the job of undermining the metaphysical principles underlying Aquinas' ways.

Robert Oerter said...

Yes, I was aware I was using "QM" as a shorthand there. As William says, it's really all about the empirical evidence, though. As a physicist, I have to take the world as I find it. Time and again, we have been misled by our preconceptions of what MUST be true about the way the universe works.

What's unscientific is to take your preferred metaphysics and force the experimental data to conform to it. The scientific approach goes the other way, from the data to the interpretation.

(Of course, if you can demonstrate that uncaused events are somehow logically incoherent, then the interpretation must not go that route. But that doesn't seem to be what you're claiming. Since you haven't explained the basis for taking Premise 1 as a metaphysical first principle, I'm not sure.)

Robert Oerter said...

A parable:

Imagine an alternate universe. This universe is perfectly causal and regular, with one exception. The exception: in a completely unpredictable and random manner, bunnies spontaneously appear. The best scientists can find no causal explanation for the bunnies. One day, Professor Boom! announces he has discovered the cause of the bunnies. He says it is cligax. Cligax is a hitherto unsuspected substance that determines exactly when and where the bunnies will appear. Prof. Boom! has written down the equation for cligax, and it predicts the bunnies will appear in a completely random and unpredictable manner, exactly in agreement with what is observed. When queried about how we know cligax exists, Prof. Boom! says we can see its existence through the bunnies that are produced. Are there any other effects of cligax? No, it is otherwise completely undetectable.

OK, quiz time: Does cligax exist? Is it a valid causal explanation of the bunnies?

BeingItself said...

"Is it a valid causal explanation of the bunnies?"

Of course!

And why does morphine put people to sleep? It's dormative virtue of course. And what makes trains go? The élan locomotif!

We have been having this argument for hundreds of years. One side just hasn't figured out how badly they have lost. Ben, go take an epistemology lesson from Molière.

Son of Ya'Kov said...

As you can see grodrigues BI is not interested in answering anybody's questions or arguing for a point of view.

He's the stereotypical Gnu'Atheist who replaces rational argument with mindless ridicule and merely worships science but doesn't actually know any real science.

I bet dollars to donuts you my good grodrigues you have forgotten more science then BI has ever learned in his lifetime or will likely learn till he leaves his parents basement.

Cheers!

BeingItself said...
This comment has been removed by the author.
BeingItself said...

Ben,

If you own TLS, just look up the examples I referred too. The copy I read was a library book so I can't give page numbers.

Feser's argument depends on the distinction between essentially ordered and accidentally ordered chains. But when he tries to give concrete examples of the former they are actually examples of the later. Feser shoots himself in the foot because his argument is based on false physics. Oops.

Son of Ya'Kov said...

>If you own TLS, just look up the examples I referred too. The copy I read was a library book so I can't give page numbers.

What did I tell ya grodrigues?

He didn't read squat.

grodrigues said...

@Robert Oerter:

"What's unscientific is to take your preferred metaphysics and force the experimental data to conform to it. The scientific approach goes the other way, from the data to the interpretation."

This is cute. Pray tell me, how am I forcing the experimental data to conform to my metaphysical view? By insisting that whatever comes to be must have a cause? How is that forcing my views on the data? How can you even *begin* to make science if you deny such a principle? Do you even realize that the complete absurdity of what you are espousing spells the *death* of the empirical sciences? If there are no causes, there is no science, because science is knowledge via causal explanations. And how one would even recognize an uncaused event to begin with? I am trying to imagine (imagine in the sense of picturing, not of conceiving with the intellect) how an uncaused event in the universe would appear to us; the best I can do is "radioactive decay" without the presence of radioactive material. It would literally be magic and outside the reach of rationality. I take that back, it is *worse* than magic. And what is cause writ large that QM is allegedly violating? This is a metaphysical question, not a physical question, so should I conclude that you are also imposing your metaphysical views on the empirical data? And what is your scientific evidence for uncaused events? There is none, zero, zilch, nada, so stop saying there is. You might as well say that there is scientific data that refutes or verifies Lindemann's theorem say (the number pi is transcendental, and in particular, irrational). All you have is an argument from ignorance: you jump from an epistemic uncertainty to a categorical metaphysical statement of absence of causality. And what is *your* favored interpretation of QM? Copenhagen Interpretation? Assuming it is, pray tell us, what is the empirical evidence for the categorical claim that the state vector is not an observable? What are the exact definitions of "observer" and "measurement process" that CI so conveniently defines only in the most vague way? What should one make out of observer and measurement process in Quantum Gravity when the system under study is the whole universe? Shall we posit a classic observer outside the universe? Invoke God? What?

Really, have you been paying attention? I have just repeated what I and other people have been saying all along. I can keep doing it, but it is somewhat jarring you know. Go learn some philosophy and what the issues of contention really are and then we can have a proper talk.

note: as far as your parable, I do not have patience for silly gnuisms. You respond your own stupid question. And by the way, go read Anscombe demolishing Hume on "conceiving" events without a cause.

grodrigues said...

@BeingItself:

Prof. Feser introduces the ordered per se and ordered per accidens distinction in the third chapter, section "Existence of God", subsection "The unmoved mover", page 91. He introduces it using the examples of a father begetting sons (ordered per accidens), a potter and a hand pushing a stone via a stick (ordered per se). Later (page 95) he further illustrates the argument with an infinite series of cabooses or a brush with a handle of infinite length. The only thing resembling physics is the discussion of Newton's first law (pages 101 and 102), but no mistakes there.

So there, I have done the work that you were unable to do, and I have hardly found any physics there, much less any blunder, and certainly not hilarious. So my conclusion is that... hmmm... how shall I put it *really* gently... you have not been telling the truth.

If I have missed something, then do the work and inform me. Still waiting for the famous mathematical blunders.

And I am laughing. But it is not at Prof. Feser.

BeingItself said...

That was what I was thinking of. The hand pushing a stick pushing a rock. He said the cause (the movement of the hand) occurred simultaneously with the movement of the rock. That's just false.

Hands and sticks and rocks are physical things, no? That is physics.

grodrigues, give me an example in nature of an essentially ordered causal chain, as defined my Feser.

Son of Ya'Kov said...

That's Unbeguiled silly objection almost verbatim.

http://edwardfeser.blogspot.com/2009/07/beguiled-by-scientism.html

You are so full of shit BI. You didn't read the book. You can't even come up with any original objections.

Son of Ya'Kov said...

Here's Unbeguiled meltdown after being refuted.

http://edwardfeser.blogspot.com/2009/07/unbeguiled-unhinged-and-unworthy-of.html

@BI

>grodrigues, give me an example in nature of an essentially ordered causal chain, as defined my Feser.

I can do that.

Someone named Ben(not moi FYI) wrote in the comments section of Feser's blog.

If one truly wanted a Newtonian essentially ordered series, one could look to action-reaction pairs, which are indeed simultaneous in the instantaneous sense.

Wow BI you really suck at this.

Maybe if you actually read the material? Or read credible critics (aka Kenny)?

Fail!

Son of Ya'Kov said...

@grodrigues

don't be too hard on Robert Oerter. He has shown good will and has admitted he doesn't know enough philosophy and could stand to know more.

For example.

QUOTE"Thanks for the responses, guys. I think this goes a long way to explaining our difference of opinion about the burden of proof. I was thinking of Premise 1 as an empirical claim. Maybe because my physicist brain reads it automatically as such.

I don't make any claims to philosophical sophistication, and of course I don't expect you to clear up all my confusions in this thread."END QUOTE

He's not BI that's for sure. We must encourage none Gnu behavior as much as possible since it's better for everybody.

Cheers.

grodrigues said...

@BeingItself:

"That was what I was thinking of. The hand pushing a stick pushing a rock. He said the cause (the movement of the hand) occurred simultaneously with the movement of the rock."

Just reread the passage (pages 92, 93) and what you say is literally false. At least with respect to the hand, stick, etc. example. He does say in the previous page that the immediate (in the logical sense) efficient cause of a thing is simultaneous with it. But this just means that in some of the illustrations of causal series he may have missed some of the intermediate links. So is this your example of an hilarious gaffe? What it is is a lack of reading skills (if you indeed ever read the book) and trying to clutch to minutia ultimately irrelevant to the arguments in a desperate attempt to disparage what you do not understand.

Not that you will understand this, but just so you know the crucial point is not simultaneity, but the instrumental nature of the chain of causality in which no member has independent causal power but only insofar as it is a link in the chain. Simultaneity makes it easier to illustrate the point. In fact, Prof. Feser even has a blog post about with the example of a stick passing through a time gate to respond to just such missing-the-point objections.

But you spoke of blunders, plural. Hilarious which presupposes a real, egregious ignorance. Until now, even if I granted a mistake (which I do not), you still have neither.

Still waiting for the mathematical blunders.

And still laughing. And not at Prof. Feser.

rank sophist said...

Imagine an alternate universe. This universe is perfectly causal and regular, with one exception. The exception: in a completely unpredictable and random manner, bunnies spontaneously appear. The best scientists can find no causal explanation for the bunnies. One day, Professor Boom! announces he has discovered the cause of the bunnies. He says it is cligax. Cligax is a hitherto unsuspected substance that determines exactly when and where the bunnies will appear. Prof. Boom! has written down the equation for cligax, and it predicts the bunnies will appear in a completely random and unpredictable manner, exactly in agreement with what is observed. When queried about how we know cligax exists, Prof. Boom! says we can see its existence through the bunnies that are produced. Are there any other effects of cligax? No, it is otherwise completely undetectable.

First, as grodrigues said, you cannot imagine something "uncaused". In fact, the Anscombe argument that he cited (a deathpunch to this kind of Humean reasoning, I might add) at one point uses rabbits as an example:

For if I say I can imagine a rabbit coming into being without a parent rabbit, well and good: I can imagine a rabbit coming into being, and our observing that there is no parent rabbit about. But what am I to imagine if I imagine a rabbit coming into being without a cause? Well, I just imagine a rabbit coming into being. That this is the imagination of a rabbit coming into being without a cause is nothing but, as it were, the title of the picture. Indeed I can form an image and give my picture that title. But from my being able to do that, nothing whatever follows about what is possible to suppose 'without contradiction or absurdity' as holding in reality.

This is a very brief excerpt from her article "Whatever Has a Beginning of Existence Must Have a Cause": Hume's Argument Exposed. It is considered by many philosophers to be a powerful refutation of Hume's causation argument. In essence, she states that it is in principle impossible to imagine something appearing "without a cause". Feser writes about this at length in TLS. I found the relevant pages excerpted here: http://insightscoop.typepad.com/2004/2011/05/the-argument-from-contingency.html?cid=6a00d83451b7c369e201538ec16453970b#comment-6a00d83451b7c369e201538ec16453970b I recommend that you read that before speaking further on the matter.

Also, your "parable" is ridiculous and completely disconnected from the subject of act/potency. Aristotle and Aquinas did not propose empirical explanations of events via some kind of undetectable principle. Act/potency is a metaphysical system. People these days love to conflate empirical science with metaphysics, but the idea is false and absurd.

In any case, you're still stuck with a "[blank]-of-the-gaps" argument, as basically everyone here has said. For the third time: it's the same argument used by intelligent design proponents and Paley followers. The same argument that Dawkins and co. ridicule incessantly, and with good reason. It is terrible. Stop using it. It makes you look bad.

Robert Oerter said...

OK, I've started reading Feser. So far (2 chapters) it's a complete disaster, but I'll keep plugging.

I'll read the Anscombe article, too, but for now I just want to point out that my description of the situation in my parable didn't make ANY metaphysical claims, only observational statements. (I mean apart from Prof. Boom's theory, which does make metaphysical claims.) I wrote that the bunnies appear "completely unpredictable and random manner", and that "The best scientists can find no causal explanation for the bunnies." I'm only talking about what's observed and the failure to find any explanation.

Now, you could try to claim the situation as I described it is logically impossible - good luck with that. Or you could say it's physically impossible, which is true (energy is not conserved) but irrelevant. You seem to be saying that it's metaphysically impossible; but no one has as yet tried to explain to me how that works.

Son of Ya'Kov said...

Well it's your own fault Robert you where told to jump to Chapter 4 unless you like right wing politics.

grodrigues said...

@BenYachov:

"Well it's your own fault Robert you where told to jump to Chapter 4 unless you like right wing politics."

I did instruct Robert Oerter to jump to chapter 4, but that was in respect to Garrigou-Lagrange's "Thomistic Synthesis" not the TLS -- and simply because the first three chapters are irrelevant to the philosophical issues.

Son of Ya'Kov said...

Nevermind then.

Son of Ya'Kov said...

>I'll read the Anscombe article, too, but for now I just want to point out that my description of the situation in my parable didn't make ANY metaphysical claims, only observational statements.

That's technically not true thought I don't doubt your sincerity.

"Imagine an alternate universe. This universe is perfectly causal and regular, with one exception."

Your reflexively using Hume's metaphysics, philosophical methodology and assumptions.

It's so ingrained you don't even know you are doing it.

"Imagining" an un-caused event? Classic Hume!

As even Dennett said "There is no such thing as philosophy-free science; there is only science whose philosophical baggage is taken on board without examination."

You need to examine your philosophical baggage Robert.

Cheers.

Robert Oerter said...

One more thing. The interpretation of quantum events as uncaused events is not the only one available, and I'm not sure if it is the right interpretation. It is, however, the STANDARD interpretation, held by a majority of physicists for the last 100 years. So when you say that interpretation has been ruled out by metaphysical considerations, you are showing your ignorance of the last century of modern physics and philosophy of physics.

You can say that all these physicists and philosophers are wrong (and hopefully explain why), or that they've missed some crucial point (and preferably explain what that point is), but do you really want to say they're all just stupid?

Stop saying that, it makes you look bad.
;)

rank sophist said...

OK, I've started reading Feser. So far (2 chapters) it's a complete disaster, but I'll keep plugging.

Like Ben said, ignore the politics. TLS was positively reviewed by Sir Anthony Kenny, an extremely important (agnostic) contemporary philosopher and one of the foremost critics of the system Feser is defending. If you can put the politics out of your mind, there's a lot to learn from that book.

I'll read the Anscombe article, too, but for now I just want to point out that my description of the situation in my parable didn't make ANY metaphysical claims, only observational statements.

Actually, observational statements were the one thing you didn't make. What you described was an imaginary scenario--almost the same imaginary scenario outlined by Anscombe. As Ben said, it's unconscious Humean reasoning. It proves nothing.

(I mean apart from Prof. Boom's theory, which does make metaphysical claims.)

You're conflating empirical science with metaphysics again. A physical theory (the thing that you proposed) is not a metaphysical system. They might look similar in some respects, I admit; but assuming that they're the same is a mistake. Physical theories are separate from and lower than metaphysics on the hierarchy. That our observations of nature yield any truth at all is itself a metaphysical presupposition. For example, prominent philosophers like Parmenides and Descartes (among others) have believed that the senses could not necessarily be trusted to give true information. Contemporary scientists are lucky that their predecessors didn't buy it.

You can say that all these physicists and philosophers are wrong (and hopefully explain why), or that they've missed some crucial point (and preferably explain what that point is), but do you really want to say they're all just stupid?

Stop saying that, it makes you look bad.


Just to clarify: I have no comment on QM interpretations, so I'm not sure why you're quoting me. Grodrigues seems to be the expert on Copenhagen here. And anyway, I don't think he was calling anyone stupid.

Son of Ya'Kov said...

>One more thing. The interpretation of quantum events as uncaused events is not the only one available,

I suspect as grodrigues has pointed out you are equating the Copenhagen interpretation with the metaphysical claim that it's un-caused.

>but do you really want to say they're all just stupid?

Anti-philosophy Atheist scientists like Krauss, Atkins, Myers or Dawkins are objectively stupid because of their rejection of philosophy and mindless embrace of Scientism.

Scientism is no better than a warmed over Logical Positivism that AG Flew at the height of his Atheism abandoned during the 50's because the who enterprise was hopelessly self-referential.

Hume was a brilliant sophist as Anscombe pointed out, not stupid.

Thought Atheist philosopher David Stove called Hume and Popper "irrationalists".

cheers.

Finish the book.

grodrigues said...

@Robert Oerter:

"The interpretation of quantum events as uncaused events is not the only one available, and I'm not sure if it is the right interpretation. It is, however, the STANDARD interpretation, held by a majority of physicists for the last 100 years. So when you say that interpretation has been ruled out by metaphysical considerations, you are showing your ignorance of the last century of modern physics and philosophy of physics."

The Copenhagen interpretation is indeed the interpretation that most physicists adopt but this does not mean much. In part, it is an historical consequence of the fact that positivism and Kant were the major philosophical influences at the time when quantum mechanics was being forged. N. Bohr is the most prominent case, although to be fair to him, his thought is not completely straightforward. Another factor, is that CI is adopted most of the times unconsciously, by osmosis as it were, during the forming years, and since there is no motivation to look for anything else it just lingers on as the default position. Physicists, if nothing else, are very practical people and they just do not care about such philosophical, foundational issues and prefer instead to pursue, you know, *physics*. Even more so today, when extreme specialization and philosophical ignorance is the rule.

But CI does *not* imply a commitment to uncaused events. I am very much surprised by your claim that the idea of uncaused events has been held by the majority of physicists and philosophers of science for the past 100 years. My experience (admittedly of only a tiny and biased sample) is that physicists prefer to remain silent and proclaim their agnosticism when the issue of uncaused events come up. But maybe you can provide some sort of evidence for your claim? Say, a serious, cogent, published philosophical defense of the idea?

Robert Oerter said...

Well, I'm still working on Feser, but so far there's almost no defense of "Whatever is changed is changed by another." The relevant pages, as far as I have read, are 54-55. Here's what he says.

"Consider also that if a potential could actualize itself, there would be no way to explain why it does so at one time rather than another."

But this is exactly what we have in the case of a quantum jump: no way to explain why it happens at one time rather than another. So to use this as an argument that every change has something outside it that changes it, is to beg the question.

"The ball melts and becomes gooey when you heat it. Why did this potential gooeyness become actual just at that point? The obvious answer is that the heat was needed to actualize it....

So, no potential can actualize itself, and in this sense anything that changes requires something outside it to change it."

This is so pathetic it can't even be called a logical fallacy. "One A has B, therefore all A have B." Maybe we can give it new name: the fallacy of induction from one case.

(I know Feser isn't this stupid, so maybe this is one of those points he's going to come back to. If so, please let me know where and I'll jump to that section. Otherwise, I'll get there eventually.)

As far as the Anscombe article, she is rebutting an argument of Hume's that purports to rebut the argument in favor of "Whatever is changed is changed by another." So even if she is right, that just mean's Hume's rebuttal is wrong. It doesn't prove that Premise 1 is right.

So, so far you guys are zero for three. I've read what you recommended, and there hasn't been any cogent defense of Premise 1. (I haven't gotten to Parmenides yet, if you have a link that would be appreciated.)

It's time to put up or shut up. Either tell me what the argument actually is, or admit that you don't have one.

Robert Oerter said...

I meant to mention there's a rebuttal of Anscombe here:
http://analysis.oxfordjournals.org/content/35/2/57.extract

BeingItself said...

grodrigues, give me an example in nature of an essentially ordered causal chain, as defined my Feser.

grodrigues said...

@Robert Oerter:

"It's time to put up or shut up. Either tell me what the argument actually is, or admit that you don't have one"

From the above linked article by A. Moreno:

""It is certain to the intellect and obvious to the senses that something is moved in this world. But whatever is moved is moved by another; for nothing is moved except insofar as it is capable of possessing the term to which it is moved, while a thing moves another inasmuch as it is actually effective. To move means to draw a thing from potentiality to actuality, and nothing can be drawn from potentiality to actuality except by something active; just as an actually hot body, like fire, makes wood, which is potentially hot, to be actually hot, thereby moving and changing it. It is not possible that a thing be both actual and potential possessor of the same term, but only of different terms; for what is actually hot cannot at the same time be potentially hot, although it can be potentially cold. Therefore it is impossible that anything be at the same time and in the same respect both mover and moved, or that it move itself. Thus whatever is moved is moved by something else."

As seen above, St. Thomas demonstrates the principle by recourse to the principle of contradiction; that is, the principle is an immediate consequence of the definition of motion."

So the principle follows from the definition of change, the primary division in being, that between potency and act, and the fact that that which is in potency cannot actualize itself. So what do you want to attack? The division in being between potency and act? The defense is in TLS. That what is in potency cannot actualize itself? Right there in TLS.

"So, so far you guys are zero for three."

As opposed to you that contents himself with repeating the same argument from ignorance over and over and already refuted, over and over?

Quantum events are uncaused. Fine. There are macroscopic quantum phenomena (Bose-Einstein condensates, superconductivity, etc.) but is every macroscopic event uncaused? If not, at what threshold of reality does causality switch on? The question almost answers itself. If yes, and this seems to be what you hold since you said previously that all chains of causality are like this (and then the mention of chains of causality is, strictly speaking, incoherent, since there is no such thing as a cause, and you have to translate everything into Humean silly-talk), then causality is not an objective fact of reality but an artifact of our descriptions. So it is not a matter of objective reality that you, Robert Oerter, is the efficient cause of the post I am responding to. So tell me, why am I answering *you* in the first place?

grodrigues said...

@BeingItself:

"grodrigues, give me an example in nature of an essentially ordered causal chain, as defined my Feser."

I doubt that you understand what is an essentially ordered causal chain, buy either way, sorry, not falling for your dodging and deflection.

By now it is obvious that you have no example of the oh so famous hilarious blunders. So as far as I am concerned, this discussion is over.

BeingItself said...

Coward.

I understand what an essentially ordered causal series is as defined by Feser. And there just ain't no such thing, as his examples make clear.

This is how bad his examples are. Suppose I said "a mammal is covered in hair and gives birth to live young, while in contrast a bird is covered in feathers and lays eggs."

And then for an example of a mammal I showed you chicken laying an egg.

TLS is chock full of idiotic stuff just like that. And you guys think it's a good book. Pathetic.

rank sophist said...

It's time to put up or shut up. Either tell me what the argument actually is, or admit that you don't have one.

So 2,500 years of philosophy developed by some of the most brilliant minds in history turns out to have no supporting argument? Are you joking? Even if none of the people here were capable of answering your questions, that would do nothing to weaken the case for act/potency, causality, etc.

Anyway, let's look at your argument.

Well, I'm still working on Feser, but so far there's almost no defense of "Whatever is changed is changed by another." The relevant pages, as far as I have read, are 54-55. Here's what he says.

"Consider also that if a potential could actualize itself, there would be no way to explain why it does so at one time rather than another."


"I'm reading a dense book on philosophy and the part I want to see isn't here yet, therefore the book is bad."

And no, the relevant pages are not 54-55. That's a brief sketch of the concept. He returns to it throughout the book. If you had actually read the TLS excerpt I posted above, you might have realized that.

But this is exactly what we have in the case of a quantum jump: no way to explain why it happens at one time rather than another. So to use this as an argument that every change has something outside it that changes it, is to beg the question.

No, actually, it isn't. It might be an inconsistency (it isn't that, either), but it is not begging the question.

In any case, what you're saying is that, because we do not understand why certain quantum events happen, the principle of causality is wrong. First, as grodrigues said, that's an argument from ignorance. Second, if the principle is wrong, that means that any event, quantum or otherwise, could happen for absolutely no reason. In such a world, science (the study of causes) becomes impossible. The use of your keyboard is no more likely to generate letters on your monitor than it is to cause flowers to sprout from your head.

"The ball melts and becomes gooey when you heat it. Why did this potential gooeyness become actual just at that point? The obvious answer is that the heat was needed to actualize it....

So, no potential can actualize itself, and in this sense anything that changes requires something outside it to change it."

This is so pathetic it can't even be called a logical fallacy. "One A has B, therefore all A have B." Maybe we can give it new name: the fallacy of induction from one case.


What you're doing could be called a fallacy, though. It's called the argument from personal incredulity.

Feser was giving ONE example. He was not making the case that rubber balls prove the existence of act/potency in all things. I mean, seriously. Do you expect him to apply the four causes and act/potency to everything in the world?

(I know Feser isn't this stupid, so maybe this is one of those points he's going to come back to. If so, please let me know where and I'll jump to that section. Otherwise, I'll get there eventually.)

Again, you should have read the excerpt I posted.

In any case, don't go into a philosophy book expecting answers to jump out at you on the first page.

(Continued)

rank sophist said...

As far as the Anscombe article, she is rebutting an argument of Hume's that purports to rebut the argument in favor of "Whatever is changed is changed by another." So even if she is right, that just mean's Hume's rebuttal is wrong. It doesn't prove that Premise 1 is right.

Give me another argument besides Hume's to support the claim that things could change without being changed by something else. Said argument cannot a "[blank]-of-the-gaps" proposition related to some as yet unexplained physical event.

So, so far you guys are zero for three. I've read what you recommended, and there hasn't been any cogent defense of Premise 1.

Again, "I am reading difficulty philosophy books and the answers have not jumped out at me. Therefore, the answers do not exist."

(I haven't gotten to Parmenides yet, if you have a link that would be appreciated.)

http://plato.stanford.edu/entries/parmenides/

William said...

"Quantum events are uncaused. Fine. There are macroscopic quantum phenomena (Bose-Einstein condensates, superconductivity, etc.) but is every macroscopic event uncaused? If not, at what threshold of reality does causality switch on?"

This is a sorites argument, and so needs no other refutation. Vagueness exists, and so do distictions.


I think we are dealing with argument by definition here. I think that the assertion the Thomists are making is that they think the need for a cause for any and all change can be seen a priori in the idea of change.

This means that we are talking past one another, since Robert and I clearly don't define "change" as rs. and gr. do.

grodrigues said...

@BeingItself:

"Coward."

Oh I am shot through the heart! A liar calling me a coward!

grodrigues said...

@William:

""Quantum events are uncaused. Fine. There are macroscopic quantum phenomena (Bose-Einstein condensates, superconductivity, etc.) but is every macroscopic event uncaused? If not, at what threshold of reality does causality switch on?"

This is a sorites argument, and so needs no other refutation. Vagueness exists, and so do distictions."

You would be right if vagueness, if any there is, was not in *language* but in the nature of reality, which is what the dispute is about (an absurd suggestion). You would be right if the vagueness response has dissolved all the Sorites' paradoxes in such a way as needing no further argument (it has not), if there was any vagueness in the notion of causality (there is not), or even if "events have causes" is a case of a soritical term (unargued, just claimed). You are very adamant that there are no causes at the fundamental, quantum level. Quantum mechanics does not switch off when passing a certain threshold as *all* reality is quantum mechanical so there are no causes, period. Above a certain threshold, of number of particles say, classical physics *approximates* quantum mechanics well enough that in practice there is no difference, foisting on us the illusion of causality, but reality is still quantum mechanical. If at the fundamental level, all events are uncaused, and all macroscopic events are the aggregate result of a collection of quantum events, the event is still uncaused. Causality does not pop up magically into reality if we just add enough particles.

BeingItself said...

grodrigues,

Suppose you flip a fair coin one trillion times. You will discover two patterns as you flip. The ratio of head:tails gets closer and closer to one, and the difference between heads and tails tends to get larger.

What causes these patterns?

William said...

Some quantum events are uncaused, but the system which includes those uncaused events still may have a cause. This is similar to Aquinas' saying that an infinite train of events needs a cause to exist at all.

Even if all causation is ultimately probabalistic, most macro-events happen reliably enough, within the scales of human history. If A causes B every time it has been seen, and would cause B all but 1 time in ten trillion years, then I believe that A causes B, period :).

Son of Ya'Kov said...

BeingItself,

You need to go f*** yourself because the troll act isn't convincing.

We have William and Robert Oerter who are the skeptics, Agnostic and Atheists dealing with their side of the issue in good faith.

You are just here to be a prick & not contribute anything of value to either side.

So really you need to F*** off for the sake of Atheists and Theists everywhere.

grodrigues said...

@William:

"Even if all causation is ultimately probabalistic"

According to you there are no causes at the quantum level, so I do not know how that can be construed as probabilistic. Not that the sentence has any meaning in the first place, because "probabilistic" is a predicate of physical theories (QM, statistical mechanics, quantum or classical, etc.) not of reality.

You systematically confuse the map (physical theories) for the territory (reality) and you show no awareness of the metaphysical issues involved.

"If A causes B every time it has been seen, and would cause B all but 1 time in ten trillion years, then I believe that A causes B, period :)."

Since you have not refuted anything and offered no arguments, just a "maybe" and a "belief", I do not feel compelled to add anything to what I already said.

grodrigues said...

@BeingItself:

"Suppose you flip a fair coin one trillion times. You will discover two patterns as you flip. The ratio of head:tails gets closer and closer to one, and the difference between heads and tails tends to get larger.

What causes these patterns?"

I do not understand the "difference" sentence as in my reading it contradicts the (true) "ratio" sentence.

Anyway, the tossing of a coin can, with sufficient accuracy, be described by the classical mechanics of rigid bodies in a fluid. But the problem is too difficult to be solved exactly, so we make the phenomenological assumption that the coin is fair (this assumption can be justified in a few ways -- read a book on statistical mechanics) and treat the problem probabilistically.

You are such a moron, that you chose an example where the indeterminacy is not in the nature of reality but is epistemic, that is, the details of the problem are too complicated to be solved exactly, and even if the problem could be solved exactly, hardly any meaningful information could be extracted from the solution, so we make some simplifying assumptions and change the questions we ask to actually get at quantitative predictions that can be falsified.

BeingItself said...

"I do not understand the "difference" sentence as in my reading it contradicts the (true) "ratio" sentence."

HAHAHAHAHA!

Here is your math and science guru! Does not understand the distinction between subtraction and division.

I'll walk you through it, genius. The "difference" is what you get when you subtract heads from tails.

Sorry for assuming you finished third grade math.

William said...

grod:

I think that you are a priori correct based on your definition of cause, but our definitions of efficient cause are just really really really different, sorry.

Robert Oerter said...

rank: You'll note that I specifically stated that the passage I quoted was likely not the whole argument. And I specifically asked where the real argument was to be found. So let's leave out the "bad reader" cuts, OK?

rank wrote, "And no, the relevant pages are not 54-55. That's a brief sketch of the concept. He returns to it throughout the book. If you had actually read the TLS excerpt I posted above, you might have realized that."

And this isn't it. I have read those pages, and in those pages Feser does nothing to support Premise 1. In fact, those pages refer to a completely different argument, the First Cause argument. We have been discussing the Unmoved Mover argument. It is about the possibility of change, not about something coming into being.

rank wrote, "Give me another argument besides Hume's..."

Look, you guys are claiming that Premise 1 is a metaphysical principle. I'm asking you to support that claim. It's no good trying to shift the burden to me and require me to DISPROVE it.

rank wrote, "Second, if the principle is wrong, that means that any event, quantum or otherwise, could happen for absolutely no reason."

I'm trying to figure out how you get your conclusion from your premise. "Some events are uncaused, therefore all events are uncaused." No, it's even worse than that, "Some events are uncaused, therefore anything can happen and logic goes out the window." Can you fill that argument out for me?

It's YOUR position that a "[blank]-of-the-gaps" argument. "I can't tell you what the cause is in quantum transitions but I know there must be one because I've defined cause in such a way that all transitions have causes."

Robert Oerter said...

grod, Thanks for pointing me back to Moreno & Aquinas. I was confused when I first read the passage you quoted because I didn't think Aquinas was saying what Moreno said he was saying. I'm still not sure.

The central point is

"To move means to draw a thing from potentiality to actuality, and nothing can be drawn from potentiality to actuality except by something active"

But here's where I'm having problems. Potentiality, according to Feser, is defined in terms of the nature of an object. But the nature of things is something we learn from experience. Indeed, the whole concept of the nature of a thing, what properties it has, what sort of things it can and can't do, is an empirical conclusion.

So it seems to me that any statement about potentialities and actualities and their relationships is ultimately empirically based.

And I think what QM is telling us is that our intuitive notions of cause and effect, of act and potency, drawn from our everyday experience, are simply inadequate when dealing with fundamental processes. (I know that's a very inadequate response, but you can hardly expect me to solve all of the problems of QM and its interpretation here!)

rank sophist said...

And this isn't it. I have read those pages, and in those pages Feser does nothing to support Premise 1. In fact, those pages refer to a completely different argument, the First Cause argument. We have been discussing the Unmoved Mover argument. It is about the possibility of change, not about something coming into being.

They are extremely similar arguments, but you have a point. Let me try to elaborate.

If you assume that change is impossible, the senses (per Parmenides) would be mere illusions, and therefore empiricism and (obviously) science would be non-starters. Why pay attention to sensory experience when you know that it's false? To get out of this trap, you have to explain how things are capable of change. Aristotle posited act/potency. Act refers to things that are; potency to things that could arise from things that are. Without the distinction, everything merely is, and therefore cannot change (again, per Parmenides).

It is obviously nonsense to state that a potency could actualize itself. A potency is not a substance; it doesn't physically exist. How could such a thing create itself? It's impossible. As a result, it must come from something that is already actual, ad infinitum.

If you want to disagree with act/potency, then you have to propose a replacement. You might consider the atomism of Democritus, since a slightly revised version of that is in vogue these days. According to that system, there is some kind of indestructible, indivisible unit of matter that lies at the bottom of everything. Things that we see are again largely illusory, but the tiny particles exist and move around, with different configurations accounting for changes as we perceive them. However, as Oderberg has written, our best scientific knowledge tells us that change occurs "all the way down". Even quarks change. So where is this indivisible, unchanging unit of matter whose clusters assume different configurations?

So I'm not sure where you're going to go from there. Change exists, and something has to account for it.

Look, you guys are claiming that Premise 1 is a metaphysical principle. I'm asking you to support that claim. It's no good trying to shift the burden to me and require me to DISPROVE it.

Above, I outlined the reason why the burden of proof is on you. I hope it's clearer now.

I'm trying to figure out how you get your conclusion from your premise. "Some events are uncaused, therefore all events are uncaused." No, it's even worse than that, "Some events are uncaused, therefore anything can happen and logic goes out the window." Can you fill that argument out for me?

Consider it for a second. If change does not necessarily follow from a cause in all cases, then it is by definition no longer a necessary truth. Hume denied that cause and effect were necessarily connected, and this resulted in a position that can only be defined as irrationalism. He himself posits that, for example, a brick thrown at a window might do nothing at all--or, as Feser says, it might turn into a bouquet of flowers. Why is one more likely than the other? There is no answer.

Hume ended up more-or-less denying the possibility of knowledge, by the way. There is no science in a Humean world, and therefore no study or prediction of quantum events.

It's YOUR position that a "[blank]-of-the-gaps" argument. "I can't tell you what the cause is in quantum transitions but I know there must be one because I've defined cause in such a way that all transitions have causes."

Actually, my position is that either A) all change is an illusion, B) reality is irrational chaos without laws, or C) Aristotle was right. Aristotle's system fits best with common sense, and, in a nice bonus, it allows for the possibility of science. Other systems don't.

Son of Ya'Kov said...

@Robert,

I don't doubt your sincerity here but I think you still
think like an empiricist scientist and not like a philosopher.

I think your are reading Feser's book as some type of
scientific text that will give you some experimental
data and not as philosophy. I am skeptical you fully
understand the difference between science and philosophy.
I firmly believe based on reason alone that even if we
lived in a Godless Universe philosophy would be needed
along with science to have a complete picture of natural
knowledge. The "science alone sans philosophy" meme of certain
fundamentalist Atheists who happen to be scientists (like Dawkins, Atkins or Kruass)
is a fundametally irrational worldview even if God doesn't exist.

Anyway these links might help you.

Blinded by Scientism
http://www.thepublicdiscourse.com/2010/03/1174

Recovering Sight after Scientism
http://www.thepublicdiscourse.com/2010/03/1184

Cheers.

grodrigues said...
This comment has been removed by the author.
grodrigues said...

@BeingItself:

"I'll walk you through it, genius. The "difference" is what you get when you subtract heads from tails."

For the record, and to set up the context, the original sentence is:

"Suppose you flip a fair coin one trillion times. You will discover two patterns as you flip. The ratio of head:tails gets closer and closer to one, and the difference between heads and tails tends to get larger."

The ratio sentence means,

heads / tails -> 1

where heads and tails are really functions depending on n, the number of tosses. A straightforward algebraic manipulation with limits implies that

heads - tails -> 0

so the difference between heads and tails does *NOT* tend to "get larger" as you assert but actually approaches zero -- precisely what one expects in a fair coin.

Since this is such an obvious contradiction, I, in a spirit of charitable reading, thought was reading you wrong. But it seems I was not, you are indeed an ignoramus, a first-rate moron, a stupid git and a liar. So from now on I will respond you only if it gives me the opportunity to have a good laugh at your expense.

And btw: HAHAHAHAHA!

grodrigues said...

@William:

"I think that you are a priori correct based on your definition of cause, but our definitions of efficient cause are just really really really different, sorry."

So pray tell us, what is your definition of efficient cause? It must be a very bizarre one indeed if it is "really really really different" from the AT one, which is just the philosophical systematization of the common sense notion of (efficient) cause. And it cannot be anything that scientists use, since what they use is the ordinary, common sense notion of cause (otherwise we would not be having all this QM talk).

grodrigues said...

@Robert Oerter:

"Potentiality, according to Feser, is defined in terms of the nature of an object."

Potency is *not* "defined in terms of the nature of an object". The specific potencies of an object do depend on the nature of the object.

"But the nature of things is something we learn from experience."

Correct.

"Indeed, the whole concept of the nature of a thing, what properties it has, what sort of things it can and can't do, is an empirical conclusion. "

According to Thomists all human knowledge starts with sense experience, so in that sense, yes, it is an empirical conclusion.

"So it seems to me that any statement about potentialities and actualities and their relationships is ultimately empirically based."

If you are using empirical in the very thin sense above, yes. As rank sophist explained quite clearly, the division between potency and act is posited to harmonize the data of the senses, things change, with the fundamental laws of being, identity and contradiction.

"And I think what QM is telling us is that our intuitive notions of cause and effect, of act and potency, drawn from our everyday experience, are simply inadequate when dealing with fundamental processes."

Make the argument.

I remind you that the problem is not a problem of QM, or even a problem of the interpretation of QM, but a metaphysical problem having to do with the nature of causality. You have to step into the metaphysical battlefield where physics is of little help. So as rank sophist said what are you going to choose? Atomism? Humean silly-talk?

William said...

grod:

"A straightforward algebraic manipulation with limits implies that
heads - tails -> 0"

It this kind of reasoning that caused Aristotle to make so many in his physics.

grod:

"So pray tell us, what is your definition of efficient cause?"

I can't expect you to understand it without more ability to do math than you have shown here.

You got others to read Feser: read

Pearl (the whole book not just a review) and get back to me :)

Son of Ya'Kov said...

@William

Do you mean "It this kind of reasoning that caused Aristotle to make so many [mistakes]in his physics."?

Dude I'm the only one around here who can have shitty grammer and sppellingl.

Stop horning in on my act.;-)

:D

Cheers.

grodrigues said...

@William:

"I can't expect you to understand it without more ability to do math than you have shown here."

The computation I made is a trivial one. Would you kindly explain to me where the mistake is?

Judea Pearl's book, a book concerned with quantitative *modeling*, emphasis on the modeling, of causal networks using graph and probability theory (with a Bayesian, instead of a frequentist say, interpretation), and the body of knowledge expounded there, is irrelevant for the metaphysical matters under discussion. It does not even answer the question I posed to you (if my memory is betraying me, can you provide the reference page?). The reasons were already explained, but since you failed to understand them, repeating them is kind of a waste of time.

grodrigues said...
This comment has been removed by the author.
grodrigues said...
This comment has been removed by the author.
William said...

grod:

Forgive me if I note that we are still talking past one another, and that you continue to take this as my talking nonsense.

Aristotle and Aquinas use only part of the common sense notion of causation: its use to explain what has changed. As a way of explaining change that has already occurred, singly, incident by incident, they have no equal in common sense terms.

However, there is another common sense way of looking at causation, which is as a way of predicting the future. Here they often fail, and other notions of causation, based on the use of wisdom and experience in the voluntaristic choice of directionality to a model of reality, does rule.

Since my definition of causation is partly about prediction and yours is only about explanation, they are really really different, and so I find that I am talking past you, sorry.

grodrigues said...

@William:

There is indeed an egregious mistake in my reasoning, so my apologies for it. Although BeingItself's two statements do contradict each other, I was also wrong in asserting that the difference approaches 0. The only thing that can be said is that their (absolute) difference is bounded.

@BeingItself:

Per the above, my apologies. The statement that the difference tends to get larger is wrong, but my statement that they approach zero is also wrong. So tit for tat, I guess you are perfectly entitled to have a good laugh at me after all.

rank sophist said...

William,

I respect your position on causation as outlined above, but I would like to ask one question. You believe that there is not an unbreakable, necessary connection between cause and effect, as evidenced by your statement that probability calculations are the supreme method for predicting future events. This opens the door for the problem of induction and--since your statement comes packaged with an implicit denial of the necessary connections between the four causes--even the New Riddle (grue and bleen). My question is this: how do you perform science in a world where predictions could be inexplicably wrong (POI) and where categorization of phenomena is false and arbitrary (NROI)? It seems like an impossibility to me.

grodrigues said...

@William:

"Since my definition of causation is partly about prediction and yours is only about explanation, they are really really different, and so I find that I am talking past you, sorry."

I do not think we are really talking past each other. As an example, you yourself said that "I think that you are a priori correct based on your definition of cause, but our definitions of efficient cause are just really really really different" so you conceded one point, so that is a measure of progress -- by progress, I mean progress in the discussion, not progress in the sense that you have conceded one point so I am closer to victory.

But I do believe that you are unaware of the problems of your position, which is basically a Humean empiricist position, or some variation thereof. Looking at the preface of the Judea Pearl book, he writes (hope no mistakes in the transcription):

"Ten years ago, when I began writing Probabilistic Reasoning in Intelligent Systems (1988), I was working within the empiricist tradition. In this tradition, probabilistic relationships constitute the foundations of human knowledge, whereas causality simply provides useful ways of abbreviating and organizing intricate patterns of probabilistic relationships. Today, my view is quite different. I now take causal relationships to be the fundamental building blocks both of physical reality and of human understanding of that reality, and I regard probabilistic relationships as but the surface phenomena of the causal machinery that underlies and propels our understanding of the world."

In section 1.3.2 (Causal relationships and their stability) he says:

"This understanding of causal influence permits us to see precisely why, and in what way, causal relationships are more "stable" than probabilistic relationships. We expect such difference in stability because causal relationships are ontological, describing objective physical constraints in our world, whereas probabilistic relationships are epistemic, reflecting what we know or believe about the world."

Judea Pearl seems to reject the empiricist tradition of Hume and see causality as an *objective* feature of reality (I only skimmed the book and a long time ago at that, so if I am misrepresenting the author, please tell me). It is particularly telling is affirmation that probabilistic relationships, that is, what we measure and what the physical theories are supposed to predict, are epistemic -- although this is probably a consequence of the Bayesian interpretation of probability theory. With the parenthetical caveat above, I sincerely do not know how invoking quantitative models of causal networks can help you in propping up *your* notion of causality.

Whatever else I wanted to say, rank sophist already said it better than I could, so I will not repeat it.

William said...

"how do you perform science in a world where predictions could be inexplicably wrong (POI) and where categorization of phenomena is false and arbitrary"

Sometimes "close enough" really is close enough.

If predictions were always correct,and categorizations always correct, we could then predict the future perfectly. We cannot because:

1. We never know enough about a system to be altogether right about our model, and

2. Even if we knew all the details completely, those above mentioned spontaneous, random quantum effects are expected to introduce unavoidable imprecision to the system.

William said...

"Pearl seems to reject the empiricist tradition of Hume and see causality as an *objective* feature of reality"

I guess that I don't think Hume was right that way either. There seems to be much more structure to experience than the Humean perspective would predict.

BeingItself said...

grodrigues,

You are simply wrong about this.

From the Wikipedia article on The Law of Large Numbers: "almost surely the absolute (nominal) difference in the number of heads and tails will become large as the number of flips becomes large".

There is no contradiction in what I said. As the number of flips increases, the difference between heads and tails tends to increase, and also the ratio of heads:tails approaches one.

Stop relying on your intuitions so much. It is quite clear that your intuitions have lead you astray here, and in other matters as well.

BeingItself said...

But why is my example important? The answer is in the article.

"The LLN is important because it "guarantees" stable long-term results for random events."

Quantum random events produce stable long term results. That is what we call causality at our level of everyday observation. Science is possible in a universe that at bottom is lousy with randomness.

rank sophist said...

BI,

Coin tosses are not random events. They're completely deterministic, even though the necessary initial conditions can't all be known. See chaos theory.

rank sophist said...

William,

The NROI isn't that easy to shake off. Consider the classic scenario: inductive research tells us that all emeralds are green, and each newly discovered green emerald solidifies this position. However, there is equal evidence to show that they are grue. An emerald is grue if it is observed to been green before, say, 2015 (or three seconds from now; it's irrelevant here), but also if it is blue and not so observed. The problem is how to explain that inductive evidence shows emeralds to be green rather than grue.

I'm sure you can see where this is going. If there's just as much evidence that they're grue, then there's just as much evidence that they're gruelow, or even gruelowple, onward forever. The category expands until it contains everything. In the end, anything can be used to prove anything else. The Wikipedia article on grue and bleen covers a few of the standard objections and Goodman's responses to them. It mentions a reply by Swinburne as though it escaped the trap, but I don't believe that this is in line with standard opinion. (I may be wrong.)

Anyway, this is just one of many issues that arise if you deny necessary connections between cause and effect, formal and final cause, etc. Also, with regard to your comment on the POI, the issue is not that we don't know enough. Rather, it's that things can change at random, without any reason. To use an example inspired by Oderberg, all gold could potentially take on the physical characteristics of, say, wood. If there is no necessary connection between the essence of gold as "a metal with atomic number 79" and the physical forms it can take, then there's nothing to stop this from happening.

BeingItself said...

Rank,

Congratulations for missing the point. See probability theory.

rank sophist said...

BI,

You're using chaos, which is ultimately deterministic, to support the possibility of "true" randomness at the quantum level. This is equivocation.

The only evidence of true randomness is within QM itself, and that's debatable.

rank sophist said...

Clarification:

The LLN only applies if you assume "random" to mean "chaotic". From the Wikipedia article on "random variable":

"In probability and statistics, a random variable or stochastic variable is, roughly speaking, a variable whose value results from the measurement of a quantity that is subject to variations due to chance (i.e. randomness, in a mathematical sense)."

Examples include roulette wheels, dice and so forth. This is not randomness strictly speaking, but chaos: events that are caused by deterministic variables too numerous and complex to be measured. Probability theory steps in to allow us to quantify something that would otherwise be too complex to measure.

If quantum events really added up to a reality based on the LLN, then it would be thanks to chaos, not "true randomness". If true randomness existed at the quantum level, then the LLN could not be applied. From the article you quoted:

"This assumes that all possible die roll outcomes are known and that Black Swan events such as a die landing on edge or being struck by lightning mid-roll are not possible or ignored if they do occur."

A truly random environment would be nothing but "Black Swan" events. It isn't a matter of something changing at one time rather than another for unknown reasons, then being generalized. There would be nothing to generalize.

Events described as "truly random" are events that have not yet been understood, and uncaused-QM proponents are people who don't understand what their position entails. Also, the idea that we can officially call the case on QM randomness "closed", when we haven't even reconciled the system with relativity, is hilarious. It's the same logic that leads people to think that our current brain scanning machines should be used for public policy decisions.

William said...

rs:
The green/grue problem is a not a worry problem for anyone who is willing to admit that they may need to change their mind in the future.

In the meantime, voluntarism says the wise may stick to what seems wise to them, and use "green" not "grue" even though this may be made to look arbitrary from the weird perspective of the example.

William said...

I think that, even if it is not a true sorites argument, we have a problem with modality and composition here:

compare:

1. No events are uncaused (Newton).

2. Some events are uncaused (Copenhagen QM).

3. assertion above: If some events are uncaused then all events are uncaused
(unsupported, fallacy of partial composition, does not follow from #2).

grodrigues said...

@BeingItself:

"You are simply wrong about this."

Indeed I am, my apologies. The limit of the ratio being 1 only says the *rates* of growth are the same but they can still grow arbitrarily apart, and they do, in the sense of convergence in probability. On account of another moronic mistake, of itself enough to flunk me at analysis I, you get to have a second laugh at the expense of my foolishness.

@William:

"2. Some events are uncaused (Copenhagen QM)."

Copenhagen does not imply a commitment to uncaused events. Not that you have given any argument to accept Copenhagen in the first place.

And "some events"? Robert Oerter was explicit in asserting that all chains of causation are like that. You seem to disagree. So what are the events in QM that are caused? What is the definition of cause in the first place that allows you to make this distinction?

"3. assertion above: If some events are uncaused then all events are uncaused. (unsupported, fallacy of partial composition, does not follow from #2)."

It is not enough to say "fallacy of partial composition", you have to show that the fallacy is indeed being made. Does a finite chain (one has to define the ordering here since we cannot rely on causal relationships. Temporal ordering?) of uncaused events make up a caused event? No. You surely have given absolutely no reason why that should be so. But even if I granted your objection it would still not be enough, simply because all physical reality, macroscopic level included, *is* quantum mechanical. So if quantum events are uncaused, events are uncaused.

Suppose we have a lump of radioactive material; it emits ionized particles, a typical quantum event. Suppose there is also a counter that collects these emitted particles. Once a single ionized particle has been collected (let us assume the counter has the accuracy to do that, this is not far from the truth) a light bulb named Byron turns on. Is the turning on of Byron caused? I will assume that we both agree that yes. But under your view, in what sense was the turning on of Byron caused? Call the turning on of Byron B, the emission of an ionized particle A. I will assume we both agree that A caused B, but under your view A is itself uncaused. So in what sense can we say that B is caused? Cause is also a *be*cause, that is a mode of explanation, but under your view B is ultimately unintelligible and in fact, all reality is ultimately unintelligible. Not that you actually gave any argument to defend this view, besides an argument from ignorance, but are you willing to commit yourself to that?

And as rank sophist continually points out, if there is indeed no causality in nature at the fundamental level (now in the broad four-fold AT sense) then why is it that radioactive material emits ionized particles at a quantifiable rate and not say, flowers? If there is no necessary, causal connection between radioactive material on the one hand and emission of ionized particles on the other, then why the emission of ionized particles by radioactive material and not of flowers? In other words, whence the "laws of Nature"? On a Humean conception they are simply descriptions of regularities and it can happen that from radioactive material flowers do pop up. On the AT conception, laws of Nature are the laws of natures, so we can make sense (at least in part) of what goes on in the world, but as far as I can see, by chucking out the window causality at the fundamental, quantum level, what you are defending is that reality is ultimately unintelligible.

Son of Ya'Kov said...

What is the practical difference between the Gnus dogmatically asserting Quantum evens are "uncaused" vs Paley type ID Theists saying "God-did-it"?

I see little difference which is why Thomism is superior.

William said...

gr:

We mostly agree. The world does not stop spinning because one of its proton spontaneously decomposes.

William said...

Ben: remember that a marked difference in the meaning of cause means a marked difference in the meaning of 'uncaused'. Uncaused here does not mean uncaused in an Aquinean sense, but it means that its timing as isolated event is unpredictable as to position in time.

Son of Ya'Kov said...

>Ben: remember that a marked difference in the meaning of cause means a marked difference in the meaning of 'uncaused'.

Naturally we can go with Hume or Aristotle & I maintain Aristotle was correct and Hume was not.

>Uncaused here does not mean uncaused in an Aquinean sense, but it means that its timing as isolated event is unpredictable as to position in time.

In which case it is pointless for Robert to cite it as a "disproof" of the 1st and 2nd ways.

William said...

Ben: Here we circle back, 180 postings later, to the point at issue: it seems to Robert, and I agree, that Aquinas is making a physical as well as a metaphysical argument here, and the physics of that part of the first way is just outdated and wrong, so the premise fails to convince.

Son of Ya'Kov said...

@William
>Ben: Here we circle back, 180 postings later, to the point at issue: it seems to Robert, and I agree, that Aquinas is making a physical as well as a metaphysical argument here,

Of course and you are both wrong & committing the fallacy of equivocation with abandon! Not to mention your mutual inability to make a distinction between science vs philosophy.

Aquinas is not making an argument from physics but a metaphysical description only. You have failed to understand that.

Hume's view of causation is a metaphysical description not an argument from physics. Same with Aquinas. Get over it!

>and the physics of that part of the first way is just outdated and wrong, so the premise fails to convince.

Claiming that "change/motion =potency being reduced to actuality" is a physical description and not a metaphysical modeling of change is just daft.

You have still got scientism on the Brain William.

Tragic.

William said...

So you don't think that the claim "Whatever is Changing is Being Changed by Something Else" is in the least about physics? If not, then I agree, this has been a tragic waste of time.

rank sophist said...

The green/grue problem is a not a worry problem for anyone who is willing to admit that they may need to change their mind in the future.

The critical point about the NROI is that it can't be argued against like that. Goodman himself believes that there will never be a non-green emerald discovered. The issue is how we support green rather than grue emeralds with inductive evidence alone, and there, as far as I know, is no answer.

For the POI, though, your position provides something of an answer.

In the meantime, voluntarism says the wise may stick to what seems wise to them, and use "green" not "grue" even though this may be made to look arbitrary from the weird perspective of the example.

Invocation of voluntarism is more-or-less a retreat into the arms of irrationalism. You're simply stating that, even though there is no objective categorical system anymore (per grue), we can pretend as though there is one.

the physics of that part of the first way is just outdated and wrong, so the premise fails to convince.

You're begging the question. Whether quantum events really are or even could be uncaused is what's at issue.

The only metaphysical evidence presented thus far for the possibility of true (i.e. non-deterministic and therefore uncaused) randomness at the quantum level was by BI, and it was just thinly-disguised equivocation. If you counter by stating that there's no reason that these "random" QM events couldn't happen within boundaries, then you've posited that QM events are merely chaotic rather than truly random. An uncaused random event does not exist within a "range" of potential outcomes, because, by defintion, it cannot be a potency of some prior event. Therefore, it's either total randomness (and therefore grod's Byron and flowers examples) or merely chaotic determinism (and therefore proof of the solidity of the First Way).

Appealing to the physicists who support uncaused QM events will not help you. It's been demonstrated time and time again that many contemporary physicists are metaphysical dunces, whether they be a Hawking or a Krauss.

Son of Ya'Kov said...

Ya Killing me William!

Killing me! I expect this crap from Paps or BI (& you where doing so well).

>So you don't think that the claim "Whatever is Changing is Being Changed by Something Else" is in the least about physics?

Are you making an equivocation between it being "in the least about physics" vs it being "an argument from physics"?

Because it's in the least about physics the same way biological questions are in the least about chemistry.

But read my lips. Aristotle's faulty physics belief that an object moving from point A to B required an agent to keep it moving or it would revert to it's natural state of stasis has nothing to do with change being potency being reduced to Act.

Live with it.

William said...

Chemistry and physics as analogous to metaphysics and empirical science?

If the physicist says "according to my theory substance S will not freeze until the temperature is below 5 Kelvin" the chemist may disagree, if his testing says that it freezes at 150 Kelvin.

Do you allow physics to disagree with "Whatever is Changing is Being Changed by Something Else" or not?

Son of Ya'Kov said...

@William
>Chemistry and physics as analogous to metaphysics and empirical science?

Why is this controversial? They are not the same but clearly related toone another.

Empirical Science deals with the raw data & metaphysics deals with the philosophical modeling of the data & the analogy of being.

BTW you didn't answer my question.

Are you making an equivocation between it being "in the least about physics" vs it being "an argument from physics?

Yes? No? Maybe? I don't understand the question? etc....

Come on give me a straight answer.

>Do you allow physics to disagree with "Whatever is Changing is Being Changed by Something Else" or not?

No I don't that's on the same level as a Young Earth Creationist claiming Evolution is false because it contradicts the Second Law of Thermal Dynamics.

You can use Hume metaphysics to argue against the 1st and Second way(or try too) or you can use some other philosophy of nature(I believe Kenny makes arguments from Frege).

But enough of the catagory mistakes.

Do I have to keep posting links to Feser's essays against Scientism before you learn the difference between philosophy vs Science William?

http://edwardfeser.blogspot.com/2011/03/scientism-roundup.html

Ya Killing me!

Son of Ya'Kov said...

William I just turned 44 yesterday so I am fast becoming a grumpy old man.

I know you are trying but will you stop confusing PHILOSOPHY OF NATURE, PHYSICS AND METAPHYSICS?

Pleez while I can still enjoy my middle age?

«Oldest ‹Older   1 – 200 of 243   Newer› Newest»